[SOLVED]Tuner stoped working after kernel update (2.6.25)

I upgraded kernel 2.6.24 to 2.6.25 and my Hauppauge WINTV tuner stopped working. Although it was recognized as I show in dmesg I had the following error "tuner 1-0061: tuner type not set". I downgraded to kernel 2.6.24 (and also downgraded nvidia and lirc packages which were dependencies) and the tuner works fine. This time tuner type is set correctly "tuner 1-0061: type set to Philips PAL/SECAM m" What should I do to make it work with newest kernel?
This is some info from dmesg relevant to my tuner card.
cx88[0]: subsystem: 0070:3401, board: Hauppauge WinTV 34xxx models [card=1,autodetected]
cx88[0]: TV tuner type -1, Radio tuner type -1
cx2388x alsa driver version 0.0.6 loaded
tveeprom 1-0050: Hauppauge model 34519, rev J260, serial# 10439851
tveeprom 1-0050: tuner model is LG S001D MK3 (idx 60, type 38)
tveeprom 1-0050: TV standards PAL(B/G) PAL(I) SECAM(L/L') PAL(D/D1/K) (eeprom 0x74)
tveeprom 1-0050: audio processor is CX880 (idx 30)
tveeprom 1-0050: has radio
cx88[0]: hauppauge eeprom: model=34519
input: cx88 IR (Hauppauge WinTV 34xxx as /devices/pci0000:00/0000:00:1e.0/0000:05:00.0/input/input6
lp0: using parport0 (interrupt-driven).
cx88[0]/0: found at 0000:05:00.0, rev: 5, irq: 23, latency: 32, mmio: 0xfa000000
tuner 1-0043: chip found @ 0x86 (cx88[0])
tda9887 1-0043: tda988[5/6/7] found @ 0x43 (tuner)
tuner 1-0043: type set to tda9887
tuner 1-0061: chip found @ 0xc2 (cx88[0])
tuner-simple 1-0061: type set to 38 (Philips PAL/SECAM multi (FM1216ME MK3))
tuner 1-0061: type set to Philips PAL/SECAM m
tuner-simple 1-0061: type set to 38 (Philips PAL/SECAM multi (FM1216ME MK3))
tuner 1-0061: type set to Philips PAL/SECAM m
cx88[0]/0: registered device video0 [v4l2]
cx88[0]/0: registered device vbi0
cx88[0]/0: registered device radio0
cx88[0]/1: CX88x/0: ALSA support for cx2388x boards
NVRM: loading NVIDIA UNIX x86 Kernel Module 169.12 Thu Feb 14 17:53:07 PST 2008
Last edited by axel (2008-06-16 18:13:23)

I tried the latest 2.26.5.6 kernel and I get the same tuner 1-0061: tuner type not set message. This time when I open xawtv I can watch the first channel in my list, but I can't change between channels. Radio doesn't work at all.
Below I include modules that are loaded in kernel .25 and after that those loaded with the working .24 one. Any ideas?
kernel 2.6.25
Module Size Used by
ac 4484 0
acpi_cpufreq 7692 0
agpgart 28244 2 intel_agp,nvidia
ahci 25224 0
ata_generic 5636 0
ata_piix 17668 5
battery 10372 0
btcx_risc 4488 3 cx8800,cx88_alsa,cx88xx
button 6416 0
cdrom 33952 1 sr_mod
compat_ioctl32 1536 1 cx8800
coretemp 6400 0
cpufreq_ondemand 7180 2
crc_itu_t 2304 1 firewire_core
cx8800 28956 0
cx88_alsa 10504 1
cx88xx 59560 2 cx8800,cx88_alsa
dock 7952 1 libata
ehci_hcd 33804 0
evdev 9472 4
ext3 124040 4
fan 4356 0
ff_memless 5128 1 usbhid
firewire_core 36928 1 firewire_ohci
firewire_ohci 16512 0
freq_table 4112 2 cpufreq_ondemand,acpi_cpufreq
hid 39296 1 usbhid
hwmon_vid 3328 1 it87
i2c_algo_bit 6020 1 cx88xx
i2c_core 19348 17 tuner,tea5767,tda8290,tda18271,tda827x,tuner_xc2028,xc5000,tda9887,tuner_simple,mt20xx,tea5761,cx88xx,i2c_algo_bit,tveeprom,v4l2_common,i2c_i801,nvidia
i2c_i801 9232 0
ieee1394 79288 1 ohci1394
intel_agp 23740 0
ipv6 256196 10
ir_common 32772 1 cx88xx
it87 19856 0
jbd 44052 1 ext3
libata 142096 5 pata_acpi,ata_piix,ahci,ata_generic,pata_jmicron
lp 9444 0
mbcache 7172 1 ext3
mt20xx 12040 1 tuner
nvidia 7096356 30
ohci1394 28720 0
parport 31596 3 parport_pc,ppdev,lp
parport_pc 34884 1
pata_acpi 4992 0
pata_jmicron 4224 0
pcspkr 2816 0
ppdev 7556 0
ppp_generic 24348 0
processor 32096 2 thermal,acpi_cpufreq
r8169 25476 0
rtc_cmos 9120 0
rtc_core 15516 1 rtc_cmos
rtc_lib 2944 1 rtc_core
scsi_mod 92204 4 sg,sd_mod,sr_mod,libata
sd_mod 23320 6
sg 27188 0
slhc 6016 1 ppp_generic
snd 46628 14 cx88_alsa,snd_seq_oss,snd_seq,snd_seq_device,snd_hda_intel,snd_hwdep,snd_pcm_oss,snd_pcm,snd_timer,snd_mixer_oss
snd_hda_intel 336440 1
snd_hwdep 7428 1 snd_hda_intel
snd_mixer_oss 14848 1 snd_pcm_oss
snd_page_alloc 8072 2 snd_hda_intel,snd_pcm
snd_pcm 68228 3 cx88_alsa,snd_hda_intel,snd_pcm_oss
snd_pcm_oss 38656 0
snd_seq 48432 4 snd_seq_oss,snd_seq_midi_event
snd_seq_device 6796 2 snd_seq_oss,snd_seq
snd_seq_midi_event 6656 1 snd_seq_oss
snd_seq_oss 30336 0
snd_timer 19848 2 snd_seq,snd_pcm
soundcore 6496 1 snd
sr_mod 15300 0
tda18271 31496 1 tda8290
tda827x 10116 1 tda8290
tda8290 12676 1 tuner
tda9887 9348 1 tuner
tea5761 4868 1 tuner
tea5767 6404 1 tuner
thermal 15260 0
tuner 35296 0
tuner_simple 8712 1 tuner
tuner_xc2028 19984 1 tuner
tveeprom 14608 1 cx88xx
uhci_hcd 22288 0
usbcore 129776 4 usbhid,ehci_hcd,uhci_hcd
usbhid 42944 0
v4l1_compat 13956 1 videodev
v4l2_common 10624 2 tuner,cx8800
videobuf_core 15876 3 cx8800,cx88xx,videobuf_dma_sg
videobuf_dma_sg 11396 3 cx8800,cx88_alsa,cx88xx
videodev 31616 3 tuner,cx8800,cx88xx
xc5000 10244 1 tuner
kernel 2.6.24
Module Size Used by
ac 4612 0
acpi_cpufreq 11292 0
agpgart 28120 2 nvidia,intel_agp
ahci 23812 0
ata_generic 5380 0
ata_piix 14852 5
battery 10756 0
btcx_risc 4488 3 cx88_alsa,cx8800,cx88xx
button 6544 0
cdrom 34080 1 sr_mod
compat_ioctl32 1536 1 cx8800
cpufreq_ondemand 7180 2
crc_itu_t 2304 1 firewire_core
cx8800 29104 0
cx88_alsa 10376 1
cx88xx 58792 2 cx88_alsa,cx8800
ehci_hcd 33804 0
evdev 9344 4
ext3 124040 4
fan 3844 0
ff_memless 5128 1 usbhid
firewire_core 36800 1 firewire_ohci
firewire_ohci 16000 0
freq_table 4112 2 cpufreq_ondemand,acpi_cpufreq
hid 38272 1 usbhid
hwmon_vid 3328 1 it87
i2c_algo_bit 6020 1 cx88xx
i2c_core 19584 11 tuner,tea5767,tda8290,tuner_simple,mt20xx,tea5761,cx88xx,i2c_algo_bit,tveeprom,i2c_i801,nvidia
i2c_i801 8848 0
ieee1394 81464 1 ohci1394
intel_agp 22164 0
ipv6 253764 10
ir_common 30724 1 cx88xx
it87 18060 0
jbd 44308 1 ext3
libata 141424 5 pata_acpi,ata_piix,ahci,ata_generic,pata_jmicron
lp 9220 0
mbcache 7168 1 ext3
mt20xx 11912 1 tuner
nvidia 7817696 28
ohci1394 28976 0
parport 31560 3 parport_pc,ppdev,lp
parport_pc 35300 1
pata_acpi 5248 0
pata_jmicron 4352 0
pcspkr 2944 0
ppdev 7556 0
ppp_generic 24596 0
processor 29144 2 thermal,acpi_cpufreq
r8169 25732 0
rtc_cmos 7712 0
rtc_core 15752 1 rtc_cmos
rtc_lib 3072 1 rtc_core
sd_mod 23168 6
sg 26896 0
slhc 6016 1 ppp_generic
snd 46436 14 cx88_alsa,snd_hda_intel,snd_seq_oss,snd_seq,snd_seq_device,snd_hwdep,snd_pcm_oss,snd_pcm,snd_timer,snd_mixer_oss
snd_hda_intel 292000 1
snd_hwdep 7556 1 snd_hda_intel
snd_mixer_oss 14848 1 snd_pcm_oss
snd_page_alloc 8072 2 snd_hda_intel,snd_pcm
snd_pcm 70404 3 cx88_alsa,snd_hda_intel,snd_pcm_oss
snd_pcm_oss 38432 0
snd_seq 48464 4 snd_seq_oss,snd_seq_midi_event
snd_seq_device 6796 2 snd_seq_oss,snd_seq
snd_seq_midi_event 6656 1 snd_seq_oss
snd_seq_oss 30336 0
snd_timer 19588 2 snd_seq,snd_pcm
soundcore 6496 1 snd
sr_mod 15268 0
tda8290 11140 1 tuner
tea5761 4740 1 tuner
tea5767 5636 1 tuner
thermal 12956 0
tuner 40224 0
tuner_simple 8584 1 tuner
tveeprom 14736 1 cx88xx
uhci_hcd 23312 0
usbcore 128620 4 usbhid,uhci_hcd,ehci_hcd
usbhid 42560 0
v4l1_compat 13828 1 videodev
v4l2_common 16256 4 tuner,cx8800,cx88xx,videodev
videobuf_core 15620 3 cx8800,cx88xx,videobuf_dma_sg
videobuf_dma_sg 11524 3 cx88_alsa,cx8800,cx88xx
videodev 26624 2 cx8800,cx88xx

Similar Messages

  • Device-mapper not working after kernel update

    Hi, yesterday i've upgraded kernel to 2.6.36 and I was so happily so i rebooted immediatly, but it ended up with error. that it can't found /dev/mapper/archarray-archroot, which is, of course my root partition.
    I'm using to raid1 partitions:
    /dev/md0 with boot ext2 partitions (archboot partition)
    /dev/md1 with LVM (volume group arrcharay, partitions archroot, archvar and archdata)
    When in sandbox i've tried lsmod and i see, that i have raid1,ext2,ext4(root partition),md_mod and dm_mod loaded(and some more of course), so i think there's nothing missing to fire-up root partition.
    I've tried to boot up install (from USB stick) and mount root patition with following:
    modprobe dm_mod raid1
    mdamdm --examine --scan >> /etc/mdam.conf
    mdassemble
    vgscan
    vgchange -ay
    And it worked, so my disk nor LVM setting is not broken.
    Only thing that i think i can did wrong, is that i've commented /dev/md1 entry in /etc/mdam.conf. I was confused by "name=archiso:1" because i haven't use "archiso" anywhere when creating LVM volumes. So i thought i will try it, and if anything go bad, i will undo this. But it worked ... until kernel update.

    It was problem with my badly edited /etc/mdadm.conf, so I'm sorry for this.

  • Satellite A200: Some devices stoped working after BIOS update

    I recently updated my BIOS to V2.60 on my Sat A200
    but ever since i apdated the BIOS some of the hardware components stoped working.
    So i decided to use the product recovery CD but it wont work anymore.
    Each time during its loading stage the system crashes and i get a blue screen mentioning a file called CLFS.SYS as the root of the problem
    Is there a solution for my problem?

    Hi
    What Sat A200-xxx do you have exactly?
    From my experience there are different BIOS versions for different operating system.
    The Vista and XP needs different BIOS!
    But I wonder why the Toshiba recovery disk does not function anymore? ?:|
    Usually you should be able to recover the system no matte what BIOS has been installed.
    Did you try to format the HDD firstly using another disk for example like Win XP?
    Check this!
    I think you should format the HDD again and then should try to recover the notebook using the Recovery disk

  • [SOLVED]Two finger scrolling/tapping stoped working after xorg update

    Hi everyone, i updated my system yesterday to the new xorg server (1.11.1-2). Everything works except for the two finger scrolling on my synaptics touchpad.
    There is some info:
    Kernel:  3.0.6-2 (stock)
    Synaptics Touchpad driver: 1.5.0-1 (lastest)
    Right side scrolling works fine, two or three finger tapping doesn't work either.
    Here's my Xorg.log:
    [ 172.989]
    X.Org X Server 1.11.1
    Release Date: 2011-09-24
    [ 172.989] X Protocol Version 11, Revision 0
    [ 172.989] Build Operating System: Linux 3.0-ARCH i686
    [ 172.989] Current Operating System: Linux Xenogears 3.0-ARCH #1 SMP PREEMPT Fri Oct 7 10:11:05 UTC 2011 i686
    [ 172.989] Kernel command line: root=/dev/sda2 ro quiet fastboot init=/sbin/e4rat-preload
    [ 172.989] Build Date: 10 October 2011 08:49:15AM
    [ 172.989]
    [ 172.989] Current version of pixman: 0.22.2
    [ 172.989] Before reporting problems, check http://wiki.x.org
    to make sure that you have the latest version.
    [ 172.990] Markers: (--) probed, (**) from config file, (==) default setting,
    (++) from command line, (!!) notice, (II) informational,
    (WW) warning, (EE) error, (NI) not implemented, (??) unknown.
    [ 172.990] (==) Log file: "/var/log/Xorg.0.log", Time: Thu Oct 13 13:27:22 2011
    [ 172.990] (==) Using config directory: "/etc/X11/xorg.conf.d"
    [ 172.990] (==) No Layout section. Using the first Screen section.
    [ 172.990] (==) No screen section available. Using defaults.
    [ 172.990] (**) |-->Screen "Default Screen Section" (0)
    [ 172.990] (**) | |-->Monitor "<default monitor>"
    [ 172.990] (==) No monitor specified for screen "Default Screen Section".
    Using a default monitor configuration.
    [ 172.990] (==) Automatically adding devices
    [ 172.990] (==) Automatically enabling devices
    [ 172.991] (==) FontPath set to:
    /usr/share/fonts/misc/,
    /usr/share/fonts/TTF/,
    /usr/share/fonts/OTF/,
    /usr/share/fonts/Type1/,
    /usr/share/fonts/100dpi/,
    /usr/share/fonts/75dpi/
    [ 172.991] (==) ModulePath set to "/usr/lib/xorg/modules"
    [ 172.991] (II) The server relies on udev to provide the list of input devices.
    If no devices become available, reconfigure udev or disable AutoAddDevices.
    [ 172.991] (II) Loader magic: 0x8228580
    [ 172.991] (II) Module ABI versions:
    [ 172.991] X.Org ANSI C Emulation: 0.4
    [ 172.991] X.Org Video Driver: 11.0
    [ 172.991] X.Org XInput driver : 13.0
    [ 172.991] X.Org Server Extension : 6.0
    [ 172.992] (--) PCI:*(0:1:5:0) 1002:791f:1631:c108 rev 0, Mem @ 0xc0000000/268435456, 0xfd8f0000/65536, 0xfd700000/1048576, I/O @ 0x00008800/256
    [ 172.992] (II) Open ACPI successful (/var/run/acpid.socket)
    [ 172.992] (II) LoadModule: "extmod"
    [ 172.993] (II) Loading /usr/lib/xorg/modules/extensions/libextmod.so
    [ 172.993] (II) Module extmod: vendor="X.Org Foundation"
    [ 172.993] compiled for 1.11.1, module version = 1.0.0
    [ 172.993] Module class: X.Org Server Extension
    [ 172.993] ABI class: X.Org Server Extension, version 6.0
    [ 172.993] (II) Loading extension MIT-SCREEN-SAVER
    [ 172.993] (II) Loading extension XFree86-VidModeExtension
    [ 172.993] (II) Loading extension XFree86-DGA
    [ 172.993] (II) Loading extension DPMS
    [ 172.993] (II) Loading extension XVideo
    [ 172.993] (II) Loading extension XVideo-MotionCompensation
    [ 172.993] (II) Loading extension X-Resource
    [ 172.993] (II) LoadModule: "dbe"
    [ 172.993] (II) Loading /usr/lib/xorg/modules/extensions/libdbe.so
    [ 172.993] (II) Module dbe: vendor="X.Org Foundation"
    [ 172.993] compiled for 1.11.1, module version = 1.0.0
    [ 172.993] Module class: X.Org Server Extension
    [ 172.993] ABI class: X.Org Server Extension, version 6.0
    [ 172.994] (II) Loading extension DOUBLE-BUFFER
    [ 172.994] (II) LoadModule: "glx"
    [ 172.994] (II) Loading /usr/lib/xorg/modules/extensions/libglx.so
    [ 172.994] (II) Module glx: vendor="X.Org Foundation"
    [ 172.994] compiled for 1.11.1, module version = 1.0.0
    [ 172.994] ABI class: X.Org Server Extension, version 6.0
    [ 172.994] (==) AIGLX enabled
    [ 172.994] (II) Loading extension GLX
    [ 172.994] (II) LoadModule: "record"
    [ 172.994] (II) Loading /usr/lib/xorg/modules/extensions/librecord.so
    [ 172.994] (II) Module record: vendor="X.Org Foundation"
    [ 172.994] compiled for 1.11.1, module version = 1.13.0
    [ 172.994] Module class: X.Org Server Extension
    [ 172.994] ABI class: X.Org Server Extension, version 6.0
    [ 172.994] (II) Loading extension RECORD
    [ 172.994] (II) LoadModule: "dri"
    [ 172.994] (II) Loading /usr/lib/xorg/modules/extensions/libdri.so
    [ 172.995] (II) Module dri: vendor="X.Org Foundation"
    [ 172.995] compiled for 1.11.1, module version = 1.0.0
    [ 172.995] ABI class: X.Org Server Extension, version 6.0
    [ 172.995] (II) Loading extension XFree86-DRI
    [ 172.995] (II) LoadModule: "dri2"
    [ 172.995] (II) Loading /usr/lib/xorg/modules/extensions/libdri2.so
    [ 172.995] (II) Module dri2: vendor="X.Org Foundation"
    [ 172.995] compiled for 1.11.1, module version = 1.2.0
    [ 172.995] ABI class: X.Org Server Extension, version 6.0
    [ 172.995] (II) Loading extension DRI2
    [ 172.995] (==) Matched ati as autoconfigured driver 0
    [ 172.995] (==) Matched vesa as autoconfigured driver 1
    [ 172.995] (==) Matched fbdev as autoconfigured driver 2
    [ 172.995] (==) Assigned the driver to the xf86ConfigLayout
    [ 172.995] (II) LoadModule: "ati"
    [ 172.995] (II) Loading /usr/lib/xorg/modules/drivers/ati_drv.so
    [ 172.995] (II) Module ati: vendor="X.Org Foundation"
    [ 172.995] compiled for 1.11.0, module version = 6.14.2
    [ 172.995] Module class: X.Org Video Driver
    [ 172.995] ABI class: X.Org Video Driver, version 11.0
    [ 172.995] (II) LoadModule: "radeon"
    [ 172.996] (II) Loading /usr/lib/xorg/modules/drivers/radeon_drv.so
    [ 172.996] (II) Module radeon: vendor="X.Org Foundation"
    [ 172.996] compiled for 1.11.0, module version = 6.14.2
    [ 172.996] Module class: X.Org Video Driver
    [ 172.996] ABI class: X.Org Video Driver, version 11.0
    [ 172.996] (II) LoadModule: "vesa"
    [ 172.996] (II) Loading /usr/lib/xorg/modules/drivers/vesa_drv.so
    [ 172.996] (II) Module vesa: vendor="X.Org Foundation"
    [ 172.996] compiled for 1.10.99.902, module version = 2.3.0
    [ 172.996] Module class: X.Org Video Driver
    [ 172.996] ABI class: X.Org Video Driver, version 11.0
    [ 172.996] (II) LoadModule: "fbdev"
    [ 172.997] (II) Loading /usr/lib/xorg/modules/drivers/fbdev_drv.so
    [ 172.997] (II) Module fbdev: vendor="X.Org Foundation"
    [ 172.997] compiled for 1.10.99.902, module version = 0.4.2
    [ 172.997] ABI class: X.Org Video Driver, version 11.0
    [ 172.997] (II) RADEON: Driver for ATI Radeon chipsets:
    ********************LOTS OF VIDEO CARDS*******************************
    [ 173.004] (II) VESA: driver for VESA chipsets: vesa
    [ 173.004] (II) FBDEV: driver for framebuffer: fbdev
    [ 173.004] (++) using VT number 8
    [ 173.015] (II) Loading /usr/lib/xorg/modules/drivers/radeon_drv.so
    [ 173.015] (II) [KMS] Kernel modesetting enabled.
    [ 173.015] (WW) Falling back to old probe method for vesa
    [ 173.015] (WW) Falling back to old probe method for fbdev
    [ 173.015] (II) Loading sub module "fbdevhw"
    [ 173.015] (II) LoadModule: "fbdevhw"
    [ 173.016] (II) Loading /usr/lib/xorg/modules/libfbdevhw.so
    [ 173.016] (II) Module fbdevhw: vendor="X.Org Foundation"
    [ 173.016] compiled for 1.11.1, module version = 0.0.2
    [ 173.016] ABI class: X.Org Video Driver, version 11.0
    [ 173.016] (II) RADEON(0): Creating default Display subsection in Screen section
    "Default Screen Section" for depth/fbbpp 24/32
    [ 173.016] (==) RADEON(0): Depth 24, (--) framebuffer bpp 32
    [ 173.016] (II) RADEON(0): Pixel depth = 24 bits stored in 4 bytes (32 bpp pixmaps)
    [ 173.016] (==) RADEON(0): Default visual is TrueColor
    [ 173.016] (==) RADEON(0): RGB weight 888
    [ 173.016] (II) RADEON(0): Using 8 bits per RGB (8 bit DAC)
    [ 173.016] (--) RADEON(0): Chipset: "ATI Radeon X1200" (ChipID = 0x791f)
    [ 173.016] (II) RADEON(0): PCI card detected
    [ 173.016] drmOpenDevice: node name is /dev/dri/card0
    [ 173.016] drmOpenDevice: open result is 9, (OK)
    [ 173.016] drmOpenByBusid: Searching for BusID pci:0000:01:05.0
    [ 173.016] drmOpenDevice: node name is /dev/dri/card0
    [ 173.016] drmOpenDevice: open result is 9, (OK)
    [ 173.016] drmOpenByBusid: drmOpenMinor returns 9
    [ 173.016] drmOpenByBusid: drmGetBusid reports pci:0000:01:05.0
    [ 173.017] (II) Loading sub module "exa"
    [ 173.017] (II) LoadModule: "exa"
    [ 173.017] (II) Loading /usr/lib/xorg/modules/libexa.so
    [ 173.017] (II) Module exa: vendor="X.Org Foundation"
    [ 173.017] compiled for 1.11.1, module version = 2.5.0
    [ 173.017] ABI class: X.Org Video Driver, version 11.0
    [ 173.017] (II) RADEON(0): KMS Color Tiling: enabled
    [ 173.017] (II) RADEON(0): KMS Pageflipping: enabled
    [ 173.017] (II) RADEON(0): SwapBuffers wait for vsync: enabled
    [ 173.190] (II) RADEON(0): Output VGA-0 has no monitor section
    [ 173.190] (II) RADEON(0): Output LVDS has no monitor section
    [ 173.356] (II) RADEON(0): EDID for output VGA-0
    [ 173.356] (II) RADEON(0): EDID for output LVDS
    [ 173.356] (II) RADEON(0): Manufacturer: AUO Model: 8174 Serial#: 0
    [ 173.356] (II) RADEON(0): Year: 2006 Week: 1
    [ 173.356] (II) RADEON(0): EDID Version: 1.3
    [ 173.356] (II) RADEON(0): Digital Display Input
    [ 173.356] (II) RADEON(0): Max Image Size [cm]: horiz.: 33 vert.: 21
    [ 173.357] (II) RADEON(0): Gamma: 2.20
    [ 173.357] (II) RADEON(0): No DPMS capabilities specified
    [ 173.357] (II) RADEON(0): Supported color encodings: RGB 4:4:4 YCrCb 4:4:4
    [ 173.357] (II) RADEON(0): First detailed timing is preferred mode
    [ 173.357] (II) RADEON(0): redX: 0.590 redY: 0.345 greenX: 0.315 greenY: 0.555
    [ 173.357] (II) RADEON(0): blueX: 0.155 blueY: 0.155 whiteX: 0.313 whiteY: 0.329
    [ 173.357] (II) RADEON(0): Manufacturer's mask: 0
    [ 173.357] (II) RADEON(0): Supported detailed timing:
    [ 173.357] (II) RADEON(0): clock: 71.1 MHz Image Size: 331 x 207 mm
    [ 173.357] (II) RADEON(0): h_active: 1280 h_sync: 1328 h_sync_end 1360 h_blank_end 1440 h_border: 0
    [ 173.357] (II) RADEON(0): v_active: 800 v_sync: 803 v_sync_end 809 v_blanking: 823 v_border: 0
    [ 173.357] (II) RADEON(0): Unknown vendor-specific block f
    [ 173.357] (II) RADEON(0): AUO
    [ 173.357] (II) RADEON(0): B154EW08 V1
    [ 173.357] (II) RADEON(0): EDID (in hex):
    [ 173.357] (II) RADEON(0): 00ffffffffffff0006af748100000000
    [ 173.357] (II) RADEON(0): 01100103802115780a1cf59758508e27
    [ 173.357] (II) RADEON(0): 27505400000001010101010101010101
    [ 173.357] (II) RADEON(0): 010101010101c71b00a0502017303020
    [ 173.357] (II) RADEON(0): 36004bcf100000180000000f00000000
    [ 173.357] (II) RADEON(0): 00000000000000000020000000fe0041
    [ 173.357] (II) RADEON(0): 554f0a202020202020202020000000fe
    [ 173.357] (II) RADEON(0): 004231353445573038205631200a0043
    [ 173.357] (II) RADEON(0): Printing probed modes for output LVDS
    [ 173.357] (II) RADEON(0): Modeline "1280x800"x60.0 71.11 1280 1328 1360 1440 800 803 809 823 -hsync -vsync (49.4 kHz)
    [ 173.357] (II) RADEON(0): Modeline "1280x720"x59.9 74.50 1280 1344 1472 1664 720 723 728 748 -hsync +vsync (44.8 kHz)
    [ 173.357] (II) RADEON(0): Modeline "1152x768"x59.8 71.75 1152 1216 1328 1504 768 771 781 798 -hsync +vsync (47.7 kHz)
    [ 173.357] (II) RADEON(0): Modeline "1024x768"x59.9 63.50 1024 1072 1176 1328 768 771 775 798 -hsync +vsync (47.8 kHz)
    [ 173.357] (II) RADEON(0): Modeline "800x600"x59.9 38.25 800 832 912 1024 600 603 607 624 -hsync +vsync (37.4 kHz)
    [ 173.357] (II) RADEON(0): Modeline "848x480"x59.7 31.50 848 872 952 1056 480 483 493 500 -hsync +vsync (29.8 kHz)
    [ 173.357] (II) RADEON(0): Modeline "720x480"x59.7 26.75 720 744 808 896 480 483 493 500 -hsync +vsync (29.9 kHz)
    [ 173.357] (II) RADEON(0): Modeline "640x480"x59.4 23.75 640 664 720 800 480 483 487 500 -hsync +vsync (29.7 kHz)
    [ 173.357] (II) RADEON(0): Output VGA-0 disconnected
    [ 173.357] (II) RADEON(0): Output LVDS connected
    [ 173.357] (II) RADEON(0): Using exact sizes for initial modes
    [ 173.357] (II) RADEON(0): Output LVDS using initial mode 1280x800
    [ 173.357] (II) RADEON(0): Using default gamma of (1.0, 1.0, 1.0) unless otherwise stated.
    [ 173.357] (II) RADEON(0): mem size init: gart size :1fdff000 vram size: s:10000000 visible:fbd8000
    [ 173.357] (II) RADEON(0): EXA: Driver will allow EXA pixmaps in VRAM
    [ 173.357] (==) RADEON(0): DPI set to (96, 96)
    [ 173.357] (II) Loading sub module "fb"
    [ 173.357] (II) LoadModule: "fb"
    [ 173.358] (II) Loading /usr/lib/xorg/modules/libfb.so
    [ 173.358] (II) Module fb: vendor="X.Org Foundation"
    [ 173.358] compiled for 1.11.1, module version = 1.0.0
    [ 173.358] ABI class: X.Org ANSI C Emulation, version 0.4
    [ 173.358] (II) Loading sub module "ramdac"
    [ 173.358] (II) LoadModule: "ramdac"
    [ 173.358] (II) Module "ramdac" already built-in
    [ 173.358] (II) UnloadModule: "vesa"
    [ 173.358] (II) Unloading vesa
    [ 173.358] (II) UnloadModule: "fbdev"
    [ 173.358] (II) Unloading fbdev
    [ 173.358] (II) UnloadModule: "fbdevhw"
    [ 173.358] (II) Unloading fbdevhw
    [ 173.358] (--) Depth 24 pixmap format is 32 bpp
    [ 173.358] (II) RADEON(0): [DRI2] Setup complete
    [ 173.358] (II) RADEON(0): [DRI2] DRI driver: r300
    [ 173.359] (II) RADEON(0): Front buffer size: 4000K
    [ 173.359] (II) RADEON(0): VRAM usage limit set to 228499K
    [ 173.359] (==) RADEON(0): Backing store disabled
    [ 173.359] (II) RADEON(0): Direct rendering enabled
    [ 173.359] (II) RADEON(0): Render acceleration enabled for R300/R400/R500 type cards.
    [ 173.359] (II) RADEON(0): Setting EXA maxPitchBytes
    [ 173.359] (II) EXA(0): Driver allocated offscreen pixmaps
    [ 173.359] (II) EXA(0): Driver registered support for the following operations:
    [ 173.359] (II) Solid
    [ 173.359] (II) Copy
    [ 173.359] (II) Composite (RENDER acceleration)
    [ 173.359] (II) UploadToScreen
    [ 173.359] (II) DownloadFromScreen
    [ 173.359] (II) RADEON(0): Acceleration enabled
    [ 173.359] (==) RADEON(0): DPMS enabled
    [ 173.359] (==) RADEON(0): Silken mouse enabled
    [ 173.359] (II) RADEON(0): Set up textured video
    [ 173.359] (II) RADEON(0): RandR 1.2 enabled, ignore the following RandR disabled message.
    [ 173.359] (--) RandR disabled
    [ 173.359] (II) Initializing built-in extension Generic Event Extension
    [ 173.359] (II) Initializing built-in extension SHAPE
    [ 173.360] (II) Initializing built-in extension MIT-SHM
    [ 173.360] (II) Initializing built-in extension XInputExtension
    [ 173.360] (II) Initializing built-in extension XTEST
    [ 173.360] (II) Initializing built-in extension BIG-REQUESTS
    [ 173.360] (II) Initializing built-in extension SYNC
    [ 173.360] (II) Initializing built-in extension XKEYBOARD
    [ 173.360] (II) Initializing built-in extension XC-MISC
    [ 173.360] (II) Initializing built-in extension SECURITY
    [ 173.360] (II) Initializing built-in extension XINERAMA
    [ 173.360] (II) Initializing built-in extension XFIXES
    [ 173.360] (II) Initializing built-in extension RENDER
    [ 173.360] (II) Initializing built-in extension RANDR
    [ 173.360] (II) Initializing built-in extension COMPOSITE
    [ 173.360] (II) Initializing built-in extension DAMAGE
    [ 173.378] (II) AIGLX: enabled GLX_MESA_copy_sub_buffer
    [ 173.378] (II) AIGLX: enabled GLX_INTEL_swap_event
    [ 173.378] (II) AIGLX: enabled GLX_SGI_swap_control and GLX_MESA_swap_control
    [ 173.378] (II) AIGLX: enabled GLX_SGI_make_current_read
    [ 173.378] (II) AIGLX: GLX_EXT_texture_from_pixmap backed by buffer objects
    [ 173.379] (II) AIGLX: Loaded and initialized r300
    [ 173.379] (II) GLX: Initialized DRI2 GL provider for screen 0
    [ 173.380] (II) RADEON(0): Setting screen physical size to 338 x 211
    [ 173.458] (II) config/udev: Adding input device Power Button (/dev/input/event4)
    [ 173.458] (**) Power Button: Applying InputClass "evdev keyboard catchall"
    [ 173.458] (II) LoadModule: "evdev"
    [ 173.458] (II) Loading /usr/lib/xorg/modules/input/evdev_drv.so
    [ 173.459] (II) Module evdev: vendor="X.Org Foundation"
    [ 173.459] compiled for 1.10.99.902, module version = 2.6.0
    [ 173.459] Module class: X.Org XInput Driver
    [ 173.459] ABI class: X.Org XInput driver, version 13.0
    [ 173.459] (II) Using input driver 'evdev' for 'Power Button'
    [ 173.459] (II) Loading /usr/lib/xorg/modules/input/evdev_drv.so
    [ 173.459] (**) Power Button: always reports core events
    [ 173.459] (**) Power Button: Device: "/dev/input/event4"
    [ 173.459] (--) Power Button: Found keys
    [ 173.459] (II) Power Button: Configuring as keyboard
    [ 173.459] (**) Option "config_info" "udev:/sys/devices/LNXSYSTM:00/LNXPWRBN:00/input/input4/event4"
    [ 173.459] (II) XINPUT: Adding extended input device "Power Button" (type: KEYBOARD, id 6)
    [ 173.459] (**) Option "xkb_rules" "evdev"
    [ 173.459] (**) Option "xkb_model" "evdev"
    [ 173.459] (**) Option "xkb_layout" "us"
    [ 173.497] (II) config/udev: Adding input device Video Bus (/dev/input/event5)
    [ 173.497] (**) Video Bus: Applying InputClass "evdev keyboard catchall"
    [ 173.497] (II) Using input driver 'evdev' for 'Video Bus'
    [ 173.497] (II) Loading /usr/lib/xorg/modules/input/evdev_drv.so
    [ 173.497] (**) Video Bus: always reports core events
    [ 173.497] (**) Video Bus: Device: "/dev/input/event5"
    [ 173.497] (--) Video Bus: Found keys
    [ 173.497] (II) Video Bus: Configuring as keyboard
    [ 173.497] (**) Option "config_info" "udev:/sys/devices/LNXSYSTM:00/device:00/PNP0A03:00/device:02/LNXVIDEO:00/input/input5/event5"
    [ 173.497] (II) XINPUT: Adding extended input device "Video Bus" (type: KEYBOARD, id 7)
    [ 173.497] (**) Option "xkb_rules" "evdev"
    [ 173.497] (**) Option "xkb_model" "evdev"
    [ 173.497] (**) Option "xkb_layout" "us"
    [ 173.498] (II) config/udev: Adding input device Power Button (/dev/input/event1)
    [ 173.498] (**) Power Button: Applying InputClass "evdev keyboard catchall"
    [ 173.498] (II) Using input driver 'evdev' for 'Power Button'
    [ 173.498] (II) Loading /usr/lib/xorg/modules/input/evdev_drv.so
    [ 173.498] (**) Power Button: always reports core events
    [ 173.498] (**) Power Button: Device: "/dev/input/event1"
    [ 173.498] (--) Power Button: Found keys
    [ 173.498] (II) Power Button: Configuring as keyboard
    [ 173.498] (**) Option "config_info" "udev:/sys/devices/LNXSYSTM:00/device:00/PNP0C0C:00/input/input1/event1"
    [ 173.498] (II) XINPUT: Adding extended input device "Power Button" (type: KEYBOARD, id 8)
    [ 173.498] (**) Option "xkb_rules" "evdev"
    [ 173.498] (**) Option "xkb_model" "evdev"
    [ 173.498] (**) Option "xkb_layout" "us"
    [ 173.499] (II) config/udev: Adding input device Lid Switch (/dev/input/event3)
    [ 173.499] (II) No input driver/identifier specified (ignoring)
    [ 173.499] (II) config/udev: Adding input device Sleep Button (/dev/input/event2)
    [ 173.499] (**) Sleep Button: Applying InputClass "evdev keyboard catchall"
    [ 173.499] (II) Using input driver 'evdev' for 'Sleep Button'
    [ 173.499] (II) Loading /usr/lib/xorg/modules/input/evdev_drv.so
    [ 173.499] (**) Sleep Button: always reports core events
    [ 173.500] (**) Sleep Button: Device: "/dev/input/event2"
    [ 173.500] (--) Sleep Button: Found keys
    [ 173.500] (II) Sleep Button: Configuring as keyboard
    [ 173.500] (**) Option "config_info" "udev:/sys/devices/LNXSYSTM:00/device:00/PNP0C0E:00/input/input2/event2"
    [ 173.500] (II) XINPUT: Adding extended input device "Sleep Button" (type: KEYBOARD, id 9)
    [ 173.500] (**) Option "xkb_rules" "evdev"
    [ 173.500] (**) Option "xkb_model" "evdev"
    [ 173.500] (**) Option "xkb_layout" "us"
    [ 173.501] (II) config/udev: Adding input device CNF7050 (/dev/input/event10)
    [ 173.501] (**) CNF7050: Applying InputClass "evdev keyboard catchall"
    [ 173.501] (II) Using input driver 'evdev' for 'CNF7050'
    [ 173.501] (II) Loading /usr/lib/xorg/modules/input/evdev_drv.so
    [ 173.501] (**) CNF7050: always reports core events
    [ 173.501] (**) CNF7050: Device: "/dev/input/event10"
    [ 173.501] (--) CNF7050: Found keys
    [ 173.501] (II) CNF7050: Configuring as keyboard
    [ 173.501] (**) Option "config_info" "udev:/sys/devices/pci0000:00/0000:00:13.5/usb1/1-7/1-7:1.0/input/input10/event10"
    [ 173.501] (II) XINPUT: Adding extended input device "CNF7050" (type: KEYBOARD, id 10)
    [ 173.501] (**) Option "xkb_rules" "evdev"
    [ 173.501] (**) Option "xkb_model" "evdev"
    [ 173.501] (**) Option "xkb_layout" "us"
    [ 173.502] (II) config/udev: Adding input device HDA Digital PCBeep (/dev/input/event8)
    [ 173.502] (II) No input driver/identifier specified (ignoring)
    [ 173.502] (II) config/udev: Adding input device Asus Laptop extra buttons (/dev/input/event6)
    [ 173.502] (**) Asus Laptop extra buttons: Applying InputClass "evdev keyboard catchall"
    [ 173.502] (II) Using input driver 'evdev' for 'Asus Laptop extra buttons'
    [ 173.502] (II) Loading /usr/lib/xorg/modules/input/evdev_drv.so
    [ 173.502] (**) Asus Laptop extra buttons: always reports core events
    [ 173.502] (**) Asus Laptop extra buttons: Device: "/dev/input/event6"
    [ 173.502] (--) Asus Laptop extra buttons: Found keys
    [ 173.502] (II) Asus Laptop extra buttons: Configuring as keyboard
    [ 173.502] (**) Option "config_info" "udev:/sys/devices/platform/asus_laptop/input/input6/event6"
    [ 173.502] (II) XINPUT: Adding extended input device "Asus Laptop extra buttons" (type: KEYBOARD, id 11)
    [ 173.502] (**) Option "xkb_rules" "evdev"
    [ 173.502] (**) Option "xkb_model" "evdev"
    [ 173.502] (**) Option "xkb_layout" "us"
    [ 173.503] (II) config/udev: Adding input device AT Translated Set 2 keyboard (/dev/input/event0)
    [ 173.503] (**) AT Translated Set 2 keyboard: Applying InputClass "evdev keyboard catchall"
    [ 173.503] (II) Using input driver 'evdev' for 'AT Translated Set 2 keyboard'
    [ 173.503] (II) Loading /usr/lib/xorg/modules/input/evdev_drv.so
    [ 173.503] (**) AT Translated Set 2 keyboard: always reports core events
    [ 173.503] (**) AT Translated Set 2 keyboard: Device: "/dev/input/event0"
    [ 173.503] (--) AT Translated Set 2 keyboard: Found keys
    [ 173.503] (II) AT Translated Set 2 keyboard: Configuring as keyboard
    [ 173.503] (**) Option "config_info" "udev:/sys/devices/platform/i8042/serio0/input/input0/event0"
    [ 173.503] (II) XINPUT: Adding extended input device "AT Translated Set 2 keyboard" (type: KEYBOARD, id 12)
    [ 173.503] (**) Option "xkb_rules" "evdev"
    [ 173.503] (**) Option "xkb_model" "evdev"
    [ 173.504] (**) Option "xkb_layout" "us"
    [ 173.504] (II) config/udev: Adding input device SynPS/2 Synaptics TouchPad (/dev/input/event9)
    [ 173.504] (**) SynPS/2 Synaptics TouchPad: Applying InputClass "evdev touchpad catchall"
    [ 173.504] (**) SynPS/2 Synaptics TouchPad: Applying InputClass "touchpad catchall"
    [ 173.504] (II) LoadModule: "synaptics"
    [ 173.505] (II) Loading /usr/lib/xorg/modules/input/synaptics_drv.so
    [ 173.505] (II) Module synaptics: vendor="X.Org Foundation"
    [ 173.505] compiled for 1.11.0, module version = 1.5.0
    [ 173.505] Module class: X.Org XInput Driver
    [ 173.505] ABI class: X.Org XInput driver, version 13.0
    [ 173.505] (II) Using input driver 'synaptics' for 'SynPS/2 Synaptics TouchPad'
    [ 173.505] (II) Loading /usr/lib/xorg/modules/input/synaptics_drv.so
    [ 173.505] (**) SynPS/2 Synaptics TouchPad: always reports core events
    [ 173.505] (**) Option "Device" "/dev/input/event9"
    [ 173.716] (--) synaptics: SynPS/2 Synaptics TouchPad: x-axis range 1472 - 5472
    [ 173.716] (--) synaptics: SynPS/2 Synaptics TouchPad: y-axis range 1408 - 4448
    [ 173.716] (--) synaptics: SynPS/2 Synaptics TouchPad: pressure range 0 - 255
    [ 173.716] (--) synaptics: SynPS/2 Synaptics TouchPad: finger width range 0 - 0
    [ 173.716] (--) synaptics: SynPS/2 Synaptics TouchPad: buttons: left right
    [ 173.716] (--) synaptics: SynPS/2 Synaptics TouchPad: Vendor 0x2 Product 0x7
    [ 173.716] (**) Option "TapButton1" "1"
    [ 173.716] (**) Option "TapButton2" "2"
    [ 173.716] (**) Option "TapButton3" "3"
    [ 173.823] (--) synaptics: SynPS/2 Synaptics TouchPad: touchpad found
    [ 173.823] (**) SynPS/2 Synaptics TouchPad: always reports core events
    [ 173.930] (**) Option "config_info" "udev:/sys/devices/platform/i8042/serio4/input/input9/event9"
    [ 173.930] (II) XINPUT: Adding extended input device "SynPS/2 Synaptics TouchPad" (type: TOUCHPAD, id 13)
    [ 173.930] (**) synaptics: SynPS/2 Synaptics TouchPad: (accel) MinSpeed is now constant deceleration 2.5
    [ 173.930] (**) synaptics: SynPS/2 Synaptics TouchPad: MaxSpeed is now 1.75
    [ 173.930] (**) synaptics: SynPS/2 Synaptics TouchPad: AccelFactor is now 0.040
    [ 173.930] (**) SynPS/2 Synaptics TouchPad: (accel) keeping acceleration scheme 1
    [ 173.930] (**) SynPS/2 Synaptics TouchPad: (accel) acceleration profile 1
    [ 173.930] (**) SynPS/2 Synaptics TouchPad: (accel) acceleration factor: 2.000
    [ 173.930] (**) SynPS/2 Synaptics TouchPad: (accel) acceleration threshold: 4
    [ 173.930] (--) synaptics: SynPS/2 Synaptics TouchPad: touchpad found
    [ 173.930] (II) config/udev: Adding input device SynPS/2 Synaptics TouchPad (/dev/input/mouse0)
    [ 173.930] (II) No input driver/identifier specified (ignoring)
    [ 173.931] (II) config/udev: Adding input device PC Speaker (/dev/input/event7)
    [ 173.931] (II) No input driver/identifier specified (ignoring)
    [ 180.276] (II) RADEON(0): EDID vendor "AUO", prod id 33140
    [ 180.276] (II) RADEON(0): Printing DDC gathered Modelines:
    [ 180.276] (II) RADEON(0): Modeline "1280x800"x0.0 71.11 1280 1328 1360 1440 800 803 809 823 -hsync -vsync (49.4 kHz)
    [ 183.590] (II) RADEON(0): EDID vendor "AUO", prod id 33140
    [ 183.590] (II) RADEON(0): Printing DDC gathered Modelines:
    [ 183.590] (II) RADEON(0): Modeline "1280x800"x0.0 71.11 1280 1328 1360 1440 800 803 809 823 -hsync -vsync (49.4 kHz)
    I deteled the part where it lists all the suported video cards...
    Thanks in advance.
    Last edited by Hyugga (2011-10-20 06:46:11)

    Same problem here
    Worked like a charm before.
    /etc/X11/xorg.conf
    # NOXORGCONFEXISTED: No X.org configuration file existed when this backup was created.
    /etc/X11/xorg.conf.d/10-evdev.conf
    # Catch-all evdev loader for udev-based systems
    # We don't simply match on any device since that also adds accelerometers
    # and other devices that we don't really want to use. The list below
    # matches everything but joysticks.
    Section "InputClass"
    Identifier "evdev pointer catchall"
    MatchIsPointer "on"
    MatchDevicePath "/dev/input/event*"
    Driver "evdev"
    EndSection
    Section "InputClass"
    Identifier "evdev keyboard catchall"
    MatchIsKeyboard "on"
    MatchDevicePath "/dev/input/event*"
    Driver "evdev"
    Option "XkbLayout" "de"
    Option "XkbVariant" "nodeadkeys"
    EndSection
    Section "InputClass"
    Identifier "evdev touchpad catchall"
    MatchIsTouchpad "on"
    MatchDevicePath "/dev/input/event*"
    Driver "evdev"
    EndSection
    Section "InputClass"
    Identifier "evdev tablet catchall"
    MatchIsTablet "on"
    MatchDevicePath "/dev/input/event*"
    Driver "evdev"
    EndSection
    Section "InputClass"
    Identifier "evdev touchscreen catchall"
    MatchIsTouchscreen "on"
    MatchDevicePath "/dev/input/event*"
    Driver "evdev"
    EndSection
    /etc/X11/xorg.conf.d/10-synaptics.conf
    Section "InputClass"
    Identifier "touchpad catchall"
    Driver "synaptics"
    MatchIsTouchpad "on"
    MatchDevicePath "/dev/input/event*"
    Option "TapButton1" "1"
    Option "TapButton2" "2"
    Option "TapButton3" "3"
    EndSection
    Xorg log
    [ 90.424] (II) config/udev: Adding input device SynPS/2 Synaptics TouchPad (/dev/input/event6)
    [ 90.424] (**) SynPS/2 Synaptics TouchPad: Applying InputClass "evdev touchpad catchall"
    [ 90.424] (**) SynPS/2 Synaptics TouchPad: Applying InputClass "touchpad catchall"
    [ 90.424] (II) LoadModule: "synaptics"
    [ 90.424] (II) Loading /usr/lib/xorg/modules/input/synaptics_drv.so
    [ 90.424] (II) Module synaptics: vendor="X.Org Foundation"
    [ 90.424] compiled for 1.11.0, module version = 1.5.0
    [ 90.424] Module class: X.Org XInput Driver
    [ 90.424] ABI class: X.Org XInput driver, version 13.0
    [ 90.424] (II) Using input driver 'synaptics' for 'SynPS/2 Synaptics TouchPad'
    [ 90.424] (II) Loading /usr/lib/xorg/modules/input/synaptics_drv.so
    [ 90.424] (**) SynPS/2 Synaptics TouchPad: always reports core events
    [ 90.424] (**) Option "Device" "/dev/input/event6"
    [ 90.500] (--) synaptics: SynPS/2 Synaptics TouchPad: x-axis range 1472 - 5598
    [ 90.500] (--) synaptics: SynPS/2 Synaptics TouchPad: y-axis range 1408 - 4670
    [ 90.500] (--) synaptics: SynPS/2 Synaptics TouchPad: pressure range 0 - 255
    [ 90.500] (--) synaptics: SynPS/2 Synaptics TouchPad: finger width range 0 - 0
    [ 90.500] (--) synaptics: SynPS/2 Synaptics TouchPad: buttons: left right
    [ 90.500] (--) synaptics: SynPS/2 Synaptics TouchPad: Vendor 0x2 Product 0x7
    [ 90.500] (**) Option "TapButton1" "1"
    [ 90.500] (**) Option "TapButton2" "2"
    [ 90.500] (**) Option "TapButton3" "3"
    [ 90.532] (--) synaptics: SynPS/2 Synaptics TouchPad: touchpad found
    [ 90.532] (**) SynPS/2 Synaptics TouchPad: always reports core events
    [ 90.564] (**) Option "config_info" "udev:/sys/devices/platform/i8042/serio1/input/input6/event6"
    [ 90.564] (II) XINPUT: Adding extended input device "SynPS/2 Synaptics TouchPad" (type: TOUCHPAD, id 11)
    [ 90.564] (**) synaptics: SynPS/2 Synaptics TouchPad: (accel) MinSpeed is now constant deceleration 2.5
    [ 90.564] (**) synaptics: SynPS/2 Synaptics TouchPad: MaxSpeed is now 1.75
    [ 90.564] (**) synaptics: SynPS/2 Synaptics TouchPad: AccelFactor is now 0.038
    [ 90.564] (**) SynPS/2 Synaptics TouchPad: (accel) keeping acceleration scheme 1
    [ 90.564] (**) SynPS/2 Synaptics TouchPad: (accel) acceleration profile 1
    [ 90.564] (**) SynPS/2 Synaptics TouchPad: (accel) acceleration factor: 2.000
    [ 90.564] (**) SynPS/2 Synaptics TouchPad: (accel) acceleration threshold: 4
    [ 90.564] (--) synaptics: SynPS/2 Synaptics TouchPad: touchpad found
    [ 90.564] (II) config/udev: Adding input device SynPS/2 Synaptics TouchPad (/dev/input/mouse0)
    [ 90.564] (II) No input driver/identifier specified (ignoring)
    Last edited by akurei (2011-10-14 10:28:45)

  • [SOLVED]Sounds no longer works after kernel update (Audigy 4)

    Updated kernel along with other updates. I have an audigy 4 (detected as audigy 2, always been that way).
    It's detected if I do lspci, if I go in alsaconf it also detects it and says it should work, but if I then go in alsamixer it just comes up with my USB webcam sound mixer.
    What am I doing wrong? I can't get any sound anymore and I don't even know how to start fixing it.
    I also checked lsmod and the module was loaded (emu10k1), it should work, but it doesn't.
    Any ideas?
    Edit: Trying to launch MPD:
    speng@spengpc ~ $ mpd
    output: No "audio_output" defined in config file
    output: Attempt to detect audio output device
    output: Attempting to detect a alsa audio device
    ALSA lib pcm_dmix.c:1008:(snd_pcm_dmix_open) unable to open slave
    alsa: Error opening default ALSA device: No such file or directory
    output: Attempting to detect a oss audio device
    oss: Error opening OSS device "/dev/dsp": Invalid argument
    oss: Error opening OSS device "/dev/sound/dsp": No such file or directory
    output: Unable to detect an audio device
    Aborted
    Last edited by speng (2009-08-21 02:10:39)

    Speng, perhaps there are two sound devices on your system?.. What are the contents of /dev/snd/?
    Efish, your issue is different, you've got a different card, right? And I don't get what the issue is, exactly. /etc/modprobe.d/*sound.conf?

  • Virtualbox no longer works after kernel update solved

    and before you say.. read the wiki, use google, check the forums.. I DID..
    here's the message i got
    The VirtualBox Linux kernel driver (vboxdrv) is either not loaded or there is a permission problem with /dev/vboxdrv. Please reinstall the kernel module by executing
    '/etc/init.d/vboxdrv setup'
    as root. Users of Ubuntu, Fedora or Mandriva should install the DKMS package first. This package keeps track of Linux kernel changes and recompiles the vboxdrv kernel module if necessary.
    /etc/init.d/vboxdrv setup: command not found  <---i pasted it to prove ive googled, wiki'd, and read the forums.. so what did I miss
    I did that command under my user name, sudo, and root.. said command not found.. when of course its found, it worked before 1.i used virtual box just before the kernel upgrade, 2. why isnt this command working under root or any other way?
    i just finished setting up windows 7 with the few apps i like. please don't tell me i have to reinstall virtualbox and lose everything..
    ive tried rebooting, loggin in and out a few times.. etc
    any ideas??? why what the WIKI says does NOT work at all??
    Last edited by binskipy2u (2010-08-18 21:30:09)

    when you install virtualbox_bin package,  pacman should have told you:
    ==> Run \`/etc/rc.d/vboxdrv setup\` as root every time your kernel is
    ==> upgraded, to compile the module for the new kernel version.
    And yes, '/etc/init.d/vboxdrv setup' is likely hardcoded in virtualbox and does not reflect how your distro setup is.
    The Arch Way is to put service script to  /etc/rc.d/  instead of /etc/init.d
    also note that  the vboxdrv service script from virtualbox_bin is different from other distros
    and that /etc/rc.d/rc.vboxdrv  isn't shipped by virtualbox_bin (hence you get command not found)
    in additions, vboxdrv from virtualbox_bin  has only   "setup" function

  • [SOLVED] No sound modules after kernel update

    Hello,
    after updating my kernel, I have no sound modules. Alsaconf can't detect card. My lspci sound card:
    00:1b.0 Audio device: Intel Corporation 5 Series/3400 Series Chipset High Definition Audio (rev 06)
    My lsmod:
    Module Size Used by
    joydev 9895 0
    usbhid 35352 0
    hid 82435 1 usbhid
    nvidia 12069499 40
    broadcom 7270 0
    tg3 133684 0
    ehci_hcd 40794 0
    intel_ips 11245 0
    psmouse 55501 0
    libphy 16985 2 broadcom,tg3
    i7core_edac 16231 0
    serio_raw 4294 0
    i2c_i801 8187 0
    pcspkr 1819 0
    evdev 9530 10
    usbcore 144144 3 usbhid,ehci_hcd
    edac_core 37552 3 i7core_edac
    i2c_core 20460 2 nvidia,i2c_i801
    sg 25429 0
    iTCO_wdt 11789 0
    button 4470 0
    iTCO_vendor_support 1961 1 iTCO_wdt
    processor 24224 0
    fuse 66858 2
    ext4 375413 3
    mbcache 5881 1 ext4
    jbd2 70667 1 ext4
    crc16 1359 1 ext4
    raid0 7958 2
    md_mod 100392 3 raid0
    sr_mod 14951 0
    cdrom 36553 1 sr_mod
    sd_mod 28275 8
    pata_acpi 3376 0
    ata_piix 22077 6
    libata 166724 2 pata_acpi,ata_piix
    scsi_mod 132794 4 sg,sr_mod,sd_mod,libata
    And my kernel:
    Linux nekroman 3.1.2-1-ARCH #1 SMP PREEMPT Tue Nov 22 09:17:56 CET 2011 x86_64 Intel(R) Core(TM) i5 CPU 760 @ 2.80GHz GenuineIntel GNU/Linux
    Thanks for help.
    //edit:
    Reinstalled kernel and it works!
    Last edited by Nekroman (2011-11-25 15:08:32)

    how did you re-install the kernel?  I've never done that, and am too experiencing sound issues.
    --nixIT

  • [SOLVED] Audio not working after system update

    Hi all,
    About 4 days ago, I installed Arch Linux (full systemd) with LXDE on a Dell Inspiron Mini netbook. Everything seemed to be OK until today, when I did pacman -Syu and then installed the SLiM login manager. After rebooting there was no audio. I am not sure if this happened because of the upgrade or because of SLiM. I am pretty new to systemd, so I could be missing something very obvious. I have tried many things so far, with no success:
    1) I disabled SLiM and instead started X from the terminal at boot, and nothing happened.
    2) I have removed my username from the 'audio' group, added it again, removed it, etc... No change.
    3) I downgraded systemd, systemd-sysvcompat, and xine-lib (just in case) to their previous version and nothing happened.
    4) I seem to be detecting the sound card correctly. If I run "aplay -l" as my normal, non-root user I get the same output as the root user:
    **** List of PLAYBACK Hardware Devices ****
    card 0: Intel [HDA Intel], device 0: ALC272 Analog [ALC272 Analog]
    Subdevices: 1/1
    Subdevice #0: subdevice #0
    5) If I login with SLiM, my session seems to be setup correctly, as far as I can tell:
    $ loginctl show-session $XDG_SESSION_ID
    Id=1
    Timestamp=Mon, 2012-10-29 00:38:35 EDT
    TimestampMonotonic=14973592
    DefaultControlGroup=name=systemd:/user/vrg/1
    VTNr=7
    Display=:0.0
    Remote=no
    RemoteUser=root
    Service=slim
    Leader=274
    Audit=1
    Type=x11
    Class=user
    Active=yes
    State=active
    KillProcesses=no
    IdleHint=no
    IdleSinceHint=0
    IdleSinceHintMonotonic=0
    Name=vrg
    6) I reinstalled pulseaudio, pulseaudio-alsa, alsa-plugins, and alsa-oss... Nothing.
    7) I do not have the ~/.asoundrc file.
    Here is a full list of the packages I upgraded:
    [2012-10-28 11:19] Running 'pacman -Syy'
    [2012-10-28 11:19] synchronizing package lists
    [2012-10-28 11:19] Running 'pacman -Syu'
    [2012-10-28 11:19] synchronizing package lists
    [2012-10-28 11:19] starting full system upgrade
    [2012-10-28 11:20] upgraded linux-api-headers (3.5.5-1 -> 3.6.3-1)
    [2012-10-28 11:20] Generating locales...
    [2012-10-28 11:20] en_US.UTF-8... done
    [2012-10-28 11:20] Generation complete.
    [2012-10-28 11:21] upgraded glibc (2.16.0-4 -> 2.16.0-5)
    [2012-10-28 11:21] upgraded binutils (2.22-10 -> 2.23-1)
    [2012-10-28 11:21] upgraded bison (2.6.2-1 -> 2.6.4-1)
    [2012-10-28 11:21] upgraded coreutils (8.19-1 -> 8.20-1)
    [2012-10-28 11:21] upgraded systemd (194-4 -> 195-2)
    [2012-10-28 11:21] upgraded device-mapper (2.02.97-1 -> 2.02.98-1)
    [2012-10-28 11:21] upgraded cryptsetup (1.5.0-2 -> 1.5.1-1)
    [2012-10-28 11:21] installed nss-myhostname (0.3-3)
    [2012-10-28 11:21] warning: /etc/shadow installed as /etc/shadow.pacnew
    [2012-10-28 11:21] upgraded filesystem (2012.10-1 -> 2012.10-2)
    [2012-10-28 11:21] upgraded firefox (16.0.1-1 -> 16.0.2-1)
    [2012-10-28 11:21] upgraded gcc-libs (4.7.2-1 -> 4.7.2-2)
    [2012-10-28 11:21] upgraded gcc (4.7.2-1 -> 4.7.2-2)
    [2012-10-28 11:21] upgraded lirc-utils (1:0.9.0-32 -> 1:0.9.0-33)
    [2012-10-28 11:21] upgraded lvm2 (2.02.97-1 -> 2.02.98-1)
    [2012-10-28 11:21] upgraded mkinitcpio (0.10-1 -> 0.11.0-1)
    [2012-10-28 11:21] upgraded syslinux (4.05-7 -> 4.06-1)
    [2012-10-28 11:21] upgraded systemd-sysvcompat (194-4 -> 195-2)
    [2012-10-28 11:21] upgraded xine-lib (1.2.2-2 -> 1.2.2-3)
    More information:
    $ ls -l /dev/snd
    total 0
    drwxr-xr-x 2 root root 60 Oct 29 00:38 by-path
    crw-rw----+ 1 root root 116, 5 Oct 29 00:38 controlC0
    crw-rw----+ 1 root root 116, 4 Oct 29 00:38 hwC0D0
    crw-rw----+ 1 root root 116, 3 Oct 29 00:38 pcmC0D0c
    crw-rw----+ 1 root root 116, 2 Oct 29 01:53 pcmC0D0p
    crw------- 1 root root 116, 1 Oct 29 00:38 seq
    crw-rw----+ 1 root root 116, 33 Oct 29 00:38 timer
    $ lsmod | grep '^snd' | column -t
    snd_hda_codec_realtek 52245 1
    snd_hda_intel 22256 1
    snd_hda_codec 83677 2 snd_hda_codec_realtek,snd_hda_intel
    snd_hwdep 4746 1 snd_hda_codec
    snd_pcm 61470 2 snd_hda_codec,snd_hda_intel
    snd_page_alloc 5869 2 snd_pcm,snd_hda_intel
    snd_timer 14902 1 snd_pcm
    snd 44874 8 snd_hda_codec_realtek,snd_hwdep,snd_timer,snd_pcm,snd_hda_codec,snd_hda_intel
    If I run 'fuser -v /dev/snd/*' I get nothing (blank) and if I run 'fuser -v /dev/dsp' I get file not found. I've also done 'amixer sset Master unmute' and 'speaker-test'. Everything seems OK, except that I can't hear anything!
    Any help or suggestions that could point me in the right direction would be greatly appreciated.
    Last edited by vrg (2012-10-29 07:25:36)

    Thanks Rasi. Incidentally, I just solved the issue a few minutes ago and was about to post the answer, but yes, you are right... the device was muted. It's embarrassing, but it did not seem obvious to me that the device was muted... All the sliders seemed OK, with nice, green values between 50 and 100. Also, the LXDE volume control was unmuted with a value between 50% and 100%, and I ran the 'amixer sset Master unmute' command several times. Clearly, this command does not do the job.
    What I was missing is nicely described in the PulseAudio troubleshooting:
    If one experiences no audio output via any means while using ALSA, attempt to unmute the sound card. To do this, launch alsamixer and make sure each column has a green 00 under it (this can be toggled by pressing 'm')
    I was not pressing 'm'... I know it's on the wiki, but this did not seem to be a PulseAudio issue, which was working perfectly yesterday and was not upgraded, so the PulseAudio troubleshooting was not the first place I looked for solutions. I wonder why the device got muted in the first place.
    Thanks again for the help!

  • [solved] Problems with display after kernel update.

    After using pacman -Syu and getting the newest kernel and Nvidia drivers I experience some major problems.
    My Arch freezes when I try to run some apps. Firefox, konqueror and some others work fine, but when I type "glxinfo | grep direct" or try to execute something using wine the problems begin. Terminal doesn't return any errors (it doesn't get a chance to do so, freeze is instant).
    Edit:
    I've used script from nvidia.com
    Last edited by dcfighter (2008-08-11 00:42:51)

    I examined /etc/fonts/conf.avail and found 2 files
    70-no-bitmaps.conf and
    70-yes-bitmaps.conf
    But the content of this files the same! Defined value was set to "false"
    I delete "no" file and correct "yes" file
    <fontconfig>
    <Accept>
    <selectfont>
    <acceptfont>
    <pattern>
    <patelt><bool>true</bool></patelt>
    </pattern>
    </acceptfont>
    </selectfont>
    </fontconfig>
    After this, Helvetica fonts became more likely

  • Equium L40-17M PSL49E - Wlan stoped working after BIOS update

    Hi all,
    I have problem with my lap-top.
    The laptop is Equium L40-17M, model PSL49E-00C005KS.
    Before couple days i update my BIOS with version P20B200 (bios-20080617131309), and after update, internal wireless stop working.
    Show me device is enable but activity is off, and i can't turn on device.
    First thing what i think is "Windows" and i start recovery disk. But after recovery problem is still exist, wireless do not working.
    Then i put external USB wireless, and try to connect with router.
    Computer recognize USB but he also do not working good, therefore the problem is not in wireless (i am not sure for this), but i think that problem is it because laptop can't recognize correctly USB device.
    I want to flash bios with old BIOS but i don't have it
    I will grateful to someone who have same problem, tell me how i fix it.
    Or send me old bios for my laptop.

    My mistake, I don't explain correctly what I done.
    Yet yesterday I download driver for WLAN from Toshiba site and install it, but is no result, problem still exist. Combination FN+F8 is forbidden (no displaying icon or not working signal lamp) - its all the same laptop don't have WLAN. It is absolutely strange.
    My laptop OS is: Windows Vista premium (32bit), and first I download BIOS for Vista version 2 of course (I am not such stupid to download BIOS, before read notice), after problem I download & BIOS for XP (just for test).
    And result from test is: WLAN yet not working, when I push Fn+F8, icon for WLAN show me that device is turn off and every time is always off. No matter what button I push or what command execute. Install driver for WLAN, but its same result at - not working devices.
    After all this operation I flash BIOS, with version for Vista, recover laptop with recovery disk and install driver for WLAN. No matter, my hard working was unjustifiable, device not recognized & not working.
    May be tomorrow I try to install Linux, just for try-out.

  • [Solved] ALSA stopped working after last update.

    I'm using pekwm and a 32bit install at the moment. Volwheel will not allow me to adjust the volume. I can't run alsamixer even though it's installed. My username is a member of the audio group... And when I run alsaconf it will not recognize my sound card (HDA Intel).
    [neruson@megatron ~]$ alsamixer
    cannot open mixer: No such file or directory
    [neruson@megatron ~]$ ls /usr/bin | grep alsa
    alsa_in
    alsaloop
    alsamixer
    alsa_out
    alsaucm
    When I ran alsaconf I got this message:
    No supported PnP or PCI card found.
    Any help would be appreciated
    Last edited by Mr_ED-horsey (2011-09-01 00:53:12)

    Fixed. I reinstalled "linux" and "linux-headers" and rebooted. Everything's working now

  • I tunes wont work after i updated it...

    So I downloaded the new iTunes. And now it will not open on my computer, I don't know what to do to get it to open. Any help would be appreciated! Thanks.

    Yeah, I'm not sitting here all day either...I didn't see your post either.
    and why did you not post your resolution so someone else can benefit?!

  • [Solved] Can not build modules for VMware Player after kernel updates

    I have a problem generating the modules for VMware Player after kernel updates. I am using  VMware Player version 5.02 (without patches) and the softlink according to the Wiki.
    When I start the installation of the modules from the GUI, the program just closes. Running vmware-modconfig in the CLI generates the following log file:
    2013-04-29T09:49:27.891+01:00| vthread-3| I120: Log for VMware Workstation pid=7478 version=9.0.2 build=build-1031769 option=Release
    2013-04-29T09:49:27.891+01:00| vthread-3| I120: The process is 64-bit.
    2013-04-29T09:49:27.891+01:00| vthread-3| I120: Host codepage=UTF-8 encoding=UTF-8
    2013-04-29T09:49:27.891+01:00| vthread-3| I120: Host is Linux 3.8.10-1-ARCH 2012.11.29
    2013-04-29T09:49:27.891+01:00| vthread-3| I120: Msg_Reset:
    2013-04-29T09:49:27.891+01:00| vthread-3| I120: [msg.dictionary.load.openFailed] Cannot open file "/usr/lib/vmware/settings": Datei oder Verzeichnis nicht gefunden.
    2013-04-29T09:49:27.891+01:00| vthread-3| I120: ----------------------------------------
    2013-04-29T09:49:27.891+01:00| vthread-3| I120: PREF Optional preferences file not found at /usr/lib/vmware/settings. Using default values.
    2013-04-29T09:49:27.891+01:00| vthread-3| I120: Msg_Reset:
    2013-04-29T09:49:27.891+01:00| vthread-3| I120: [msg.dictionary.load.openFailed] Cannot open file "/root/.vmware/config": Datei oder Verzeichnis nicht gefunden.
    2013-04-29T09:49:27.891+01:00| vthread-3| I120: ----------------------------------------
    2013-04-29T09:49:27.891+01:00| vthread-3| I120: PREF Optional preferences file not found at /root/.vmware/config. Using default values.
    2013-04-29T09:49:27.891+01:00| vthread-3| W110: Logging to /tmp/vmware-root/vmware-modconfig-7478.log
    2013-04-29T09:49:27.892+01:00| vthread-3| W110: Icon name must be set.
    Interestingly, after deinstallation and reinstallation the VMWare Player works fine.
    Any hint, what's going wrong?
    Last edited by Thomas_Do (2013-08-07 07:25:51)

    Today, (Kernel  3.10.5-1, VMware Player version 5.02 with kernel patches) the modules could successful be generated from the CLI:
    vmware-modconfig --console --install-all
    Solved (somehow).

  • Automatic recompiling of kernel module after kernel update.

    Hello everyone,
    i tried to search for it but without success...
    my question is - is it possible to set it up somehow that aur package would recompile after kernel update?
    The package I would like to rebuild is rt3090 which is module for my wifi card. It is very annoying to rebuild it manually. I know that my fglrx module does it automaticaly but how to set it?
    Thank all of you for any advise.

    Thank you :-) It looks ok, I will try DKMS as soon as I get some free time. If there was any problem, I would tell you.
    BTW: I use catalyst mainly because of saving energy management, which is of course better for my netbook. My card is too new to be fully supported by the opensource driver.
    EDIT:
    I solved the situation by upgrading my system, I tried in-kernel module rt2800pci. In the previous kernels it didn't work well with my crappy ralink rt3090 wifi card, but that's been improved :-) so I do not need rebuild the package anymore even because this module isn't compatible with newest kernel.
    I also learnt how to write my own hook from archwiki (https://wiki.archlinux.org/index.php/Mkinitcpio) so if the new module didn't work I would try it.
    The final solution = replace module rt3090sta by rt2800pci.
    Thanks! Arch has very powerful community :-)
    Last edited by h@lf@rk (2012-06-02 18:34:17)

  • [SOLVED] RT2870 drops connection after pacman update

    Since last pacman upgrade my USB wireless works for just a few seconds while used; while watching transfer speed I see at first full speed and then going slower and slower and in a few secs totally stopped, without being disconnected. I have to reboot to get it working again (worked sometimes by ip link set wlan 0 down & up)
    No difference using Wicd or networkmanager.
    I have never had any issues earlier even since Kernel 3.0.
    Linksys router with standard WPA2 configuration, other computers in house connecting without problems.
    lsusb:
    D-Link System DWA-140 RangeBooster N Adapter(rev.B1) [Ralink RT2870]
    uname -r
    3.2.8-1-ARCH
    I've tried blacklisting all the kernel rt2800usb modules which supposedly have taken over specific Realtek drivers according to wiki
    https://wiki.archlinux.org/index.php/Wi … and_rt2870
    with no effect.
    Tried killing wpa_supplicant (only found one instance though) as suggested on some blogpost but no effect.
    Tried the proprietary rt2870sta driver as suggested in Wiki
      https://wiki.archlinux.org/index.php/Rt2870
    -got a working link on ra0 but it wouldn't connect to WiFi (kept asking for password).
    More debug info:
    lsmod | grep rt > lsmod.txt
    exportfs 3695 1 nfsd
    rt2800usb 14400 0
    rt2x00usb 9076 1 rt2800usb
    rt2800lib 40260 1 rt2800usb
    rt2x00lib 35183 3 rt2800lib,rt2x00usb,rt2800usb
    crc_ccitt 1363 1 rt2800lib
    mac80211 228527 3 rt2x00lib,rt2800lib,rt2x00usb
    cfg80211 172260 2 mac80211,rt2x00lib
    iTCO_vendor_support 1961 1 iTCO_wdt
    usbcore 146241 7 ehci_hcd,uhci_hcd,usbhid,rt2x00usb,rt2800usb,usblp
    dmesg
    [ 0.000000] Initializing cgroup subsys cpuset
    [ 0.000000] Initializing cgroup subsys cpu
    [ 0.000000] Linux version 3.2.8-1-ARCH (tobias@T-POWA-LX) (gcc version 4.6.2 20120120 (prerelease) (GCC) ) #1 SMP PREEMPT Mon Feb 27 21:51:46 CET 2012
    [ 0.000000] Command line: root=/dev/disk/by-uuid/ce706691-4517-477c-a42d-417ab8ffef38 loglevel=3 ro quiet resume=/dev/disk/by-uuid/5a94d96e-0fdd-425a-91c5-6fe96da349f0 nomodeset
    [ 0.000000] BIOS-provided physical RAM map:
    [ 0.000000] BIOS-e820: 0000000000000000 - 000000000009fc00 (usable)
    [ 0.000000] BIOS-e820: 000000000009fc00 - 00000000000a0000 (reserved)
    [ 0.000000] BIOS-e820: 00000000000e4000 - 0000000000100000 (reserved)
    [ 0.000000] BIOS-e820: 0000000000100000 - 00000000cff70000 (usable)
    [ 0.000000] BIOS-e820: 00000000cff70000 - 00000000cff7e000 (ACPI data)
    [ 0.000000] BIOS-e820: 00000000cff7e000 - 00000000cffd0000 (ACPI NVS)
    [ 0.000000] BIOS-e820: 00000000cffd0000 - 00000000d0000000 (reserved)
    [ 0.000000] BIOS-e820: 00000000fee00000 - 00000000fee01000 (reserved)
    [ 0.000000] BIOS-e820: 00000000fff00000 - 0000000100000000 (reserved)
    [ 0.000000] BIOS-e820: 0000000100000000 - 0000000130000000 (usable)
    [ 0.000000] NX (Execute Disable) protection: active
    [ 0.000000] DMI present.
    [ 0.000000] DMI: System manufacturer P5Q SE/P5Q SE, BIOS 0502 08/20/2008
    [ 0.000000] e820 update range: 0000000000000000 - 0000000000010000 (usable) ==> (reserved)
    [ 0.000000] e820 remove range: 00000000000a0000 - 0000000000100000 (usable)
    [ 0.000000] No AGP bridge found
    [ 0.000000] last_pfn = 0x130000 max_arch_pfn = 0x400000000
    [ 0.000000] MTRR default type: uncachable
    [ 0.000000] MTRR fixed ranges enabled:
    [ 0.000000] 00000-9FFFF write-back
    [ 0.000000] A0000-BFFFF uncachable
    [ 0.000000] C0000-DFFFF write-protect
    [ 0.000000] E0000-EFFFF write-through
    [ 0.000000] F0000-FFFFF write-protect
    [ 0.000000] MTRR variable ranges enabled:
    [ 0.000000] 0 base 000000000 mask F00000000 write-back
    [ 0.000000] 1 base 100000000 mask FE0000000 write-back
    [ 0.000000] 2 base 120000000 mask FF0000000 write-back
    [ 0.000000] 3 base 0D0000000 mask FF0000000 uncachable
    [ 0.000000] 4 base 0E0000000 mask FE0000000 uncachable
    [ 0.000000] 5 disabled
    [ 0.000000] 6 disabled
    [ 0.000000] 7 disabled
    [ 0.000000] x86 PAT enabled: cpu 0, old 0x7040600070406, new 0x7010600070106
    [ 0.000000] e820 update range: 00000000d0000000 - 0000000100000000 (usable) ==> (reserved)
    [ 0.000000] last_pfn = 0xcff70 max_arch_pfn = 0x400000000
    [ 0.000000] found SMP MP-table at [ffff8800000ff780] ff780
    [ 0.000000] initial memory mapped : 0 - 20000000
    [ 0.000000] Base memory trampoline at [ffff88000009a000] 9a000 size 20480
    [ 0.000000] init_memory_mapping: 0000000000000000-00000000cff70000
    [ 0.000000] 0000000000 - 00cfe00000 page 2M
    [ 0.000000] 00cfe00000 - 00cff70000 page 4k
    [ 0.000000] kernel direct mapping tables up to cff70000 @ 1fffa000-20000000
    [ 0.000000] init_memory_mapping: 0000000100000000-0000000130000000
    [ 0.000000] 0100000000 - 0130000000 page 2M
    [ 0.000000] kernel direct mapping tables up to 130000000 @ cff6a000-cff70000
    [ 0.000000] RAMDISK: 37d76000 - 37ff0000
    [ 0.000000] ACPI: RSDP 00000000000fb2c0 00024 (v02 ACPIAM)
    [ 0.000000] ACPI: XSDT 00000000cff70100 0005C (v01 A_M_I_ OEMXSDT 08000820 MSFT 00000097)
    [ 0.000000] ACPI: FACP 00000000cff70290 000F4 (v03 A_M_I_ OEMFACP 08000820 MSFT 00000097)
    [ 0.000000] ACPI: DSDT 00000000cff70440 0907C (v01 A0985 A0985001 00000001 INTL 20051117)
    [ 0.000000] ACPI: FACS 00000000cff7e000 00040
    [ 0.000000] ACPI: APIC 00000000cff70390 0006C (v01 A_M_I_ OEMAPIC 08000820 MSFT 00000097)
    [ 0.000000] ACPI: MCFG 00000000cff70400 0003C (v01 A_M_I_ OEMMCFG 08000820 MSFT 00000097)
    [ 0.000000] ACPI: OEMB 00000000cff7e040 00081 (v01 A_M_I_ AMI_OEM 08000820 MSFT 00000097)
    [ 0.000000] ACPI: HPET 00000000cff794c0 00038 (v01 A_M_I_ OEMHPET 08000820 MSFT 00000097)
    [ 0.000000] ACPI: OSFR 00000000cff79500 000B0 (v01 A_M_I_ OEMOSFR 08000820 MSFT 00000097)
    [ 0.000000] ACPI: SSDT 00000000cff7e5d0 00A7C (v01 DpgPmm CpuPm 00000012 INTL 20051117)
    [ 0.000000] ACPI: Local APIC address 0xfee00000
    [ 0.000000] No NUMA configuration found
    [ 0.000000] Faking a node at 0000000000000000-0000000130000000
    [ 0.000000] Initmem setup node 0 0000000000000000-0000000130000000
    [ 0.000000] NODE_DATA [000000012fffb000 - 000000012fffffff]
    [ 0.000000] [ffffea0000000000-ffffea0004bfffff] PMD -> [ffff88012b600000-ffff88012f5fffff] on node 0
    [ 0.000000] Zone PFN ranges:
    [ 0.000000] DMA 0x00000010 -> 0x00001000
    [ 0.000000] DMA32 0x00001000 -> 0x00100000
    [ 0.000000] Normal 0x00100000 -> 0x00130000
    [ 0.000000] Movable zone start PFN for each node
    [ 0.000000] early_node_map[3] active PFN ranges
    [ 0.000000] 0: 0x00000010 -> 0x0000009f
    [ 0.000000] 0: 0x00000100 -> 0x000cff70
    [ 0.000000] 0: 0x00100000 -> 0x00130000
    [ 0.000000] On node 0 totalpages: 1048319
    [ 0.000000] DMA zone: 64 pages used for memmap
    [ 0.000000] DMA zone: 5 pages reserved
    [ 0.000000] DMA zone: 3914 pages, LIFO batch:0
    [ 0.000000] DMA32 zone: 16320 pages used for memmap
    [ 0.000000] DMA32 zone: 831408 pages, LIFO batch:31
    [ 0.000000] Normal zone: 3072 pages used for memmap
    [ 0.000000] Normal zone: 193536 pages, LIFO batch:31
    [ 0.000000] ACPI: PM-Timer IO Port: 0x808
    [ 0.000000] ACPI: Local APIC address 0xfee00000
    [ 0.000000] ACPI: LAPIC (acpi_id[0x01] lapic_id[0x00] enabled)
    [ 0.000000] ACPI: LAPIC (acpi_id[0x02] lapic_id[0x01] enabled)
    [ 0.000000] ACPI: LAPIC (acpi_id[0x03] lapic_id[0x82] disabled)
    [ 0.000000] ACPI: LAPIC (acpi_id[0x04] lapic_id[0x83] disabled)
    [ 0.000000] ACPI: IOAPIC (id[0x02] address[0xfec00000] gsi_base[0])
    [ 0.000000] IOAPIC[0]: apic_id 2, version 32, address 0xfec00000, GSI 0-23
    [ 0.000000] ACPI: INT_SRC_OVR (bus 0 bus_irq 0 global_irq 2 dfl dfl)
    [ 0.000000] ACPI: INT_SRC_OVR (bus 0 bus_irq 9 global_irq 9 high level)
    [ 0.000000] ACPI: IRQ0 used by override.
    [ 0.000000] ACPI: IRQ2 used by override.
    [ 0.000000] ACPI: IRQ9 used by override.
    [ 0.000000] Using ACPI (MADT) for SMP configuration information
    [ 0.000000] ACPI: HPET id: 0x8086a301 base: 0xfed00000
    [ 0.000000] SMP: Allowing 4 CPUs, 2 hotplug CPUs
    [ 0.000000] nr_irqs_gsi: 40
    [ 0.000000] PM: Registered nosave memory: 000000000009f000 - 00000000000a0000
    [ 0.000000] PM: Registered nosave memory: 00000000000a0000 - 00000000000e4000
    [ 0.000000] PM: Registered nosave memory: 00000000000e4000 - 0000000000100000
    [ 0.000000] PM: Registered nosave memory: 00000000cff70000 - 00000000cff7e000
    [ 0.000000] PM: Registered nosave memory: 00000000cff7e000 - 00000000cffd0000
    [ 0.000000] PM: Registered nosave memory: 00000000cffd0000 - 00000000d0000000
    [ 0.000000] PM: Registered nosave memory: 00000000d0000000 - 00000000fee00000
    [ 0.000000] PM: Registered nosave memory: 00000000fee00000 - 00000000fee01000
    [ 0.000000] PM: Registered nosave memory: 00000000fee01000 - 00000000fff00000
    [ 0.000000] PM: Registered nosave memory: 00000000fff00000 - 0000000100000000
    [ 0.000000] Allocating PCI resources starting at d0000000 (gap: d0000000:2ee00000)
    [ 0.000000] Booting paravirtualized kernel on bare hardware
    [ 0.000000] setup_percpu: NR_CPUS:64 nr_cpumask_bits:64 nr_cpu_ids:4 nr_node_ids:1
    [ 0.000000] PERCPU: Embedded 28 pages/cpu @ffff88012fc00000 s82176 r8192 d24320 u524288
    [ 0.000000] pcpu-alloc: s82176 r8192 d24320 u524288 alloc=1*2097152
    [ 0.000000] pcpu-alloc: [0] 0 1 2 3
    [ 0.000000] Built 1 zonelists in Node order, mobility grouping on. Total pages: 1028858
    [ 0.000000] Policy zone: Normal
    [ 0.000000] Kernel command line: root=/dev/disk/by-uuid/ce706691-4517-477c-a42d-417ab8ffef38 loglevel=3 ro quiet resume=/dev/disk/by-uuid/5a94d96e-0fdd-425a-91c5-6fe96da349f0 nomodeset
    [ 0.000000] PID hash table entries: 4096 (order: 3, 32768 bytes)
    [ 0.000000] Checking aperture...
    [ 0.000000] No AGP bridge found
    [ 0.000000] Calgary: detecting Calgary via BIOS EBDA area
    [ 0.000000] Calgary: Unable to locate Rio Grande table in EBDA - bailing!
    [ 0.000000] Memory: 4047500k/4980736k available (4488k kernel code, 787460k absent, 145776k reserved, 4409k data, 736k init)
    [ 0.000000] SLUB: Genslabs=15, HWalign=64, Order=0-3, MinObjects=0, CPUs=4, Nodes=1
    [ 0.000000] Preemptible hierarchical RCU implementation.
    [ 0.000000] Verbose stalled-CPUs detection is disabled.
    [ 0.000000] NR_IRQS:4352 nr_irqs:712 16
    [ 0.000000] Extended CMOS year: 2000
    [ 0.000000] Console: colour VGA+ 80x25
    [ 0.000000] console [tty0] enabled
    [ 0.000000] allocated 33554432 bytes of page_cgroup
    [ 0.000000] please try 'cgroup_disable=memory' option if you don't want memory cgroups
    [ 0.000000] hpet clockevent registered
    [ 0.000000] Fast TSC calibration using PIT
    [ 0.000000] Detected 2997.032 MHz processor.
    [ 0.003337] Calibrating delay loop (skipped), value calculated using timer frequency.. 5996.81 BogoMIPS (lpj=9990106)
    [ 0.003340] pid_max: default: 32768 minimum: 301
    [ 0.003361] Security Framework initialized
    [ 0.003367] AppArmor: AppArmor disabled by boot time parameter
    [ 0.006673] Dentry cache hash table entries: 524288 (order: 10, 4194304 bytes)
    [ 0.008366] Inode-cache hash table entries: 262144 (order: 9, 2097152 bytes)
    [ 0.009152] Mount-cache hash table entries: 256
    [ 0.009284] Initializing cgroup subsys cpuacct
    [ 0.009289] Initializing cgroup subsys memory
    [ 0.009297] Initializing cgroup subsys devices
    [ 0.009298] Initializing cgroup subsys freezer
    [ 0.009300] Initializing cgroup subsys net_cls
    [ 0.009302] Initializing cgroup subsys blkio
    [ 0.009332] CPU: Physical Processor ID: 0
    [ 0.009333] CPU: Processor Core ID: 0
    [ 0.009335] mce: CPU supports 6 MCE banks
    [ 0.009342] CPU0: Thermal monitoring enabled (TM2)
    [ 0.009346] using mwait in idle threads.
    [ 0.010343] ACPI: Core revision 20110623
    [ 0.013339] ftrace: allocating 17394 entries in 69 pages
    [ 0.017049] ..TIMER: vector=0x30 apic1=0 pin1=2 apic2=-1 pin2=-1
    [ 0.050789] CPU0: Intel(R) Core(TM)2 Duo CPU E8400 @ 3.00GHz stepping 06
    [ 0.053330] APIC calibration not consistent with PM-Timer: 97ms instead of 100ms
    [ 0.053330] APIC delta adjusted to PM-Timer: 2081243 (2020688)
    [ 0.053330] Performance Events: PEBS fmt0+, Core2 events, Intel PMU driver.
    [ 0.053330] ... version: 2
    [ 0.053330] ... bit width: 40
    [ 0.053330] ... generic registers: 2
    [ 0.053330] ... value mask: 000000ffffffffff
    [ 0.053330] ... max period: 000000007fffffff
    [ 0.053330] ... fixed-purpose events: 3
    [ 0.053330] ... event mask: 0000000700000003
    [ 0.070123] NMI watchdog enabled, takes one hw-pmu counter.
    [ 0.096675] Booting Node 0, Processors #1
    [ 0.096678] smpboot cpu 1: start_ip = 9a000
    [ 0.193349] NMI watchdog enabled, takes one hw-pmu counter.
    [ 0.199992] Brought up 2 CPUs
    [ 0.199995] Total of 2 processors activated (11992.51 BogoMIPS).
    [ 0.202833] devtmpfs: initialized
    [ 0.203768] PM: Registering ACPI NVS region at cff7e000 (335872 bytes)
    [ 0.203929] print_constraints: dummy:
    [ 0.203965] NET: Registered protocol family 16
    [ 0.204054] ACPI: bus type pci registered
    [ 0.204105] PCI: MMCONFIG for domain 0000 [bus 00-ff] at [mem 0xe0000000-0xefffffff] (base 0xe0000000)
    [ 0.204107] PCI: not using MMCONFIG
    [ 0.204109] PCI: Using configuration type 1 for base access
    [ 0.204490] bio: create slab <bio-0> at 0
    [ 0.204490] ACPI: Added _OSI(Module Device)
    [ 0.204490] ACPI: Added _OSI(Processor Device)
    [ 0.204490] ACPI: Added _OSI(3.0 _SCP Extensions)
    [ 0.204490] ACPI: Added _OSI(Processor Aggregator Device)
    [ 0.204490] ACPI: EC: Look up EC in DSDT
    [ 0.206720] ACPI: Executed 1 blocks of module-level executable AML code
    [ 0.211061] ACPI: SSDT 00000000cff7e0d0 00277 (v01 DpgPmm P001Ist 00000011 INTL 20051117)
    [ 0.211260] ACPI: Dynamic OEM Table Load:
    [ 0.211262] ACPI: SSDT (null) 00277 (v01 DpgPmm P001Ist 00000011 INTL 20051117)
    [ 0.211420] ACPI: SSDT 00000000cff7e350 00277 (v01 DpgPmm P002Ist 00000012 INTL 20051117)
    [ 0.211621] ACPI: Dynamic OEM Table Load:
    [ 0.211623] ACPI: SSDT (null) 00277 (v01 DpgPmm P002Ist 00000012 INTL 20051117)
    [ 0.211740] ACPI: Interpreter enabled
    [ 0.211743] ACPI: (supports S0 S1 S3 S4 S5)
    [ 0.211758] ACPI: Using IOAPIC for interrupt routing
    [ 0.211772] PCI: MMCONFIG for domain 0000 [bus 00-ff] at [mem 0xe0000000-0xefffffff] (base 0xe0000000)
    [ 0.212438] PCI: MMCONFIG at [mem 0xe0000000-0xefffffff] reserved in ACPI motherboard resources
    [ 0.246853] ACPI: No dock devices found.
    [ 0.246855] HEST: Table not found.
    [ 0.246858] PCI: Using host bridge windows from ACPI; if necessary, use "pci=nocrs" and report a bug
    [ 0.246900] ACPI: PCI Root Bridge [PCI0] (domain 0000 [bus 00-ff])
    [ 0.246996] pci_root PNP0A08:00: host bridge window [io 0x0000-0x0cf7]
    [ 0.246998] pci_root PNP0A08:00: host bridge window [io 0x0d00-0xffff]
    [ 0.247000] pci_root PNP0A08:00: host bridge window [mem 0x000a0000-0x000bffff]
    [ 0.247001] pci_root PNP0A08:00: host bridge window [mem 0x000d0000-0x000dffff]
    [ 0.247003] pci_root PNP0A08:00: host bridge window [mem 0xd0000000-0xffffffff]
    [ 0.247012] pci 0000:00:00.0: [8086:2e20] type 0 class 0x000600
    [ 0.247043] pci 0000:00:01.0: [8086:2e21] type 1 class 0x000604
    [ 0.247071] pci 0000:00:01.0: PME# supported from D0 D3hot D3cold
    [ 0.247074] pci 0000:00:01.0: PME# disabled
    [ 0.247103] pci 0000:00:1a.0: [8086:3a37] type 0 class 0x000c03
    [ 0.247137] pci 0000:00:1a.0: reg 20: [io 0xc800-0xc81f]
    [ 0.247179] pci 0000:00:1a.1: [8086:3a38] type 0 class 0x000c03
    [ 0.247213] pci 0000:00:1a.1: reg 20: [io 0xc880-0xc89f]
    [ 0.247254] pci 0000:00:1a.2: [8086:3a39] type 0 class 0x000c03
    [ 0.247288] pci 0000:00:1a.2: reg 20: [io 0xcc00-0xcc1f]
    [ 0.247336] pci 0000:00:1a.7: [8086:3a3c] type 0 class 0x000c03
    [ 0.247354] pci 0000:00:1a.7: reg 10: [mem 0xf9fffc00-0xf9ffffff]
    [ 0.247428] pci 0000:00:1a.7: PME# supported from D0 D3hot D3cold
    [ 0.247432] pci 0000:00:1a.7: PME# disabled
    [ 0.247451] pci 0000:00:1b.0: [8086:3a3e] type 0 class 0x000403
    [ 0.247463] pci 0000:00:1b.0: reg 10: [mem 0xf9ff8000-0xf9ffbfff 64bit]
    [ 0.247516] pci 0000:00:1b.0: PME# supported from D0 D3hot D3cold
    [ 0.247519] pci 0000:00:1b.0: PME# disabled
    [ 0.247535] pci 0000:00:1c.0: [8086:3a40] type 1 class 0x000604
    [ 0.247589] pci 0000:00:1c.0: PME# supported from D0 D3hot D3cold
    [ 0.247592] pci 0000:00:1c.0: PME# disabled
    [ 0.247614] pci 0000:00:1c.5: [8086:3a4a] type 1 class 0x000604
    [ 0.247669] pci 0000:00:1c.5: PME# supported from D0 D3hot D3cold
    [ 0.247672] pci 0000:00:1c.5: PME# disabled
    [ 0.247690] pci 0000:00:1d.0: [8086:3a34] type 0 class 0x000c03
    [ 0.247724] pci 0000:00:1d.0: reg 20: [io 0xc080-0xc09f]
    [ 0.247766] pci 0000:00:1d.1: [8086:3a35] type 0 class 0x000c03
    [ 0.247800] pci 0000:00:1d.1: reg 20: [io 0xc400-0xc41f]
    [ 0.247841] pci 0000:00:1d.2: [8086:3a36] type 0 class 0x000c03
    [ 0.247875] pci 0000:00:1d.2: reg 20: [io 0xc480-0xc49f]
    [ 0.247924] pci 0000:00:1d.7: [8086:3a3a] type 0 class 0x000c03
    [ 0.247941] pci 0000:00:1d.7: reg 10: [mem 0xf9fff800-0xf9fffbff]
    [ 0.248016] pci 0000:00:1d.7: PME# supported from D0 D3hot D3cold
    [ 0.248019] pci 0000:00:1d.7: PME# disabled
    [ 0.248035] pci 0000:00:1e.0: [8086:244e] type 1 class 0x000604
    [ 0.248084] pci 0000:00:1f.0: [8086:3a18] type 0 class 0x000601
    [ 0.248181] pci 0000:00:1f.2: [8086:3a20] type 0 class 0x000101
    [ 0.248193] pci 0000:00:1f.2: reg 10: [io 0xb000-0xb007]
    [ 0.248199] pci 0000:00:1f.2: reg 14: [io 0xac00-0xac03]
    [ 0.248205] pci 0000:00:1f.2: reg 18: [io 0xa880-0xa887]
    [ 0.248212] pci 0000:00:1f.2: reg 1c: [io 0xa800-0xa803]
    [ 0.248218] pci 0000:00:1f.2: reg 20: [io 0xa480-0xa48f]
    [ 0.248224] pci 0000:00:1f.2: reg 24: [io 0xa400-0xa40f]
    [ 0.248262] pci 0000:00:1f.3: [8086:3a30] type 0 class 0x000c05
    [ 0.248274] pci 0000:00:1f.3: reg 10: [mem 0xf9fff400-0xf9fff4ff 64bit]
    [ 0.248290] pci 0000:00:1f.3: reg 20: [io 0x0400-0x041f]
    [ 0.248317] pci 0000:00:1f.5: [8086:3a26] type 0 class 0x000101
    [ 0.248328] pci 0000:00:1f.5: reg 10: [io 0xc000-0xc007]
    [ 0.248335] pci 0000:00:1f.5: reg 14: [io 0xbc00-0xbc03]
    [ 0.248341] pci 0000:00:1f.5: reg 18: [io 0xb880-0xb887]
    [ 0.248347] pci 0000:00:1f.5: reg 1c: [io 0xb800-0xb803]
    [ 0.248353] pci 0000:00:1f.5: reg 20: [io 0xb480-0xb48f]
    [ 0.248359] pci 0000:00:1f.5: reg 24: [io 0xb400-0xb40f]
    [ 0.248418] pci 0000:01:00.0: [10de:05e2] type 0 class 0x000300
    [ 0.248426] pci 0000:01:00.0: reg 10: [mem 0xfd000000-0xfdffffff]
    [ 0.248434] pci 0000:01:00.0: reg 14: [mem 0xd0000000-0xdfffffff 64bit pref]
    [ 0.248443] pci 0000:01:00.0: reg 1c: [mem 0xfa000000-0xfbffffff 64bit]
    [ 0.248449] pci 0000:01:00.0: reg 24: [io 0xdc00-0xdc7f]
    [ 0.248455] pci 0000:01:00.0: reg 30: [mem 0xfea80000-0xfeafffff pref]
    [ 0.248497] pci 0000:00:01.0: PCI bridge to [bus 01-01]
    [ 0.248499] pci 0000:00:01.0: bridge window [io 0xd000-0xdfff]
    [ 0.248502] pci 0000:00:01.0: bridge window [mem 0xfa000000-0xfeafffff]
    [ 0.248505] pci 0000:00:01.0: bridge window [mem 0xd0000000-0xdfffffff 64bit pref]
    [ 0.248536] pci 0000:00:1c.0: PCI bridge to [bus 03-03]
    [ 0.248543] pci 0000:00:1c.0: bridge window [mem 0xf8f00000-0xf8ffffff 64bit pref]
    [ 0.248590] pci 0000:02:00.0: [1969:1026] type 0 class 0x000200
    [ 0.248610] pci 0000:02:00.0: reg 10: [mem 0xfebc0000-0xfebfffff 64bit]
    [ 0.248621] pci 0000:02:00.0: reg 18: [io 0xec00-0xec7f]
    [ 0.248706] pci 0000:02:00.0: PME# supported from D3hot D3cold
    [ 0.248710] pci 0000:02:00.0: PME# disabled
    [ 0.248728] pci 0000:02:00.0: disabling ASPM on pre-1.1 PCIe device. You can enable it with 'pcie_aspm=force'
    [ 0.248736] pci 0000:00:1c.5: PCI bridge to [bus 02-02]
    [ 0.248739] pci 0000:00:1c.5: bridge window [io 0xe000-0xefff]
    [ 0.248742] pci 0000:00:1c.5: bridge window [mem 0xfeb00000-0xfebfffff]
    [ 0.248794] pci 0000:00:1e.0: PCI bridge to [bus 04-04] (subtractive decode)
    [ 0.248801] pci 0000:00:1e.0: bridge window [io 0x0000-0x0cf7] (subtractive decode)
    [ 0.248803] pci 0000:00:1e.0: bridge window [io 0x0d00-0xffff] (subtractive decode)
    [ 0.248805] pci 0000:00:1e.0: bridge window [mem 0x000a0000-0x000bffff] (subtractive decode)
    [ 0.248807] pci 0000:00:1e.0: bridge window [mem 0x000d0000-0x000dffff] (subtractive decode)
    [ 0.248809] pci 0000:00:1e.0: bridge window [mem 0xd0000000-0xffffffff] (subtractive decode)
    [ 0.248822] ACPI: PCI Interrupt Routing Table [\_SB_.PCI0._PRT]
    [ 0.248885] ACPI: PCI Interrupt Routing Table [\_SB_.PCI0.P0P2._PRT]
    [ 0.248906] ACPI: PCI Interrupt Routing Table [\_SB_.PCI0.P0P1._PRT]
    [ 0.248953] ACPI: PCI Interrupt Routing Table [\_SB_.PCI0.P0P9._PRT]
    [ 0.248985] ACPI: PCI Interrupt Routing Table [\_SB_.PCI0.P0P4._PRT]
    [ 0.249003] pci0000:00: Requesting ACPI _OSC control (0x1d)
    [ 0.249005] pci0000:00: ACPI _OSC request failed (AE_NOT_FOUND), returned control mask: 0x1d
    [ 0.249007] ACPI _OSC control for PCIe not granted, disabling ASPM
    [ 0.254308] ACPI: PCI Interrupt Link [LNKA] (IRQs 3 4 5 6 7 *10 11 12 14 15)
    [ 0.254341] ACPI: PCI Interrupt Link [LNKB] (IRQs 3 4 5 6 7 10 *11 12 14 15)
    [ 0.254373] ACPI: PCI Interrupt Link [LNKC] (IRQs 3 4 5 6 7 10 11 12 14 *15)
    [ 0.254405] ACPI: PCI Interrupt Link [LNKD] (IRQs 3 4 *5 6 7 10 11 12 14 15)
    [ 0.254437] ACPI: PCI Interrupt Link [LNKE] (IRQs 3 4 5 6 7 10 11 12 14 15) *0, disabled.
    [ 0.254469] ACPI: PCI Interrupt Link [LNKF] (IRQs 3 4 5 6 7 10 11 12 *14 15)
    [ 0.254501] ACPI: PCI Interrupt Link [LNKG] (IRQs *3 4 5 6 7 10 11 12 14 15)
    [ 0.254533] ACPI: PCI Interrupt Link [LNKH] (IRQs 3 4 5 6 *7 10 11 12 14 15)
    [ 0.254589] vgaarb: device added: PCI:0000:01:00.0,decodes=io+mem,owns=io+mem,locks=none
    [ 0.254589] vgaarb: loaded
    [ 0.254589] vgaarb: bridge control possible 0000:01:00.0
    [ 0.254589] PCI: Using ACPI for IRQ routing
    [ 0.259493] PCI: pci_cache_line_size set to 64 bytes
    [ 0.259546] reserve RAM buffer: 000000000009fc00 - 000000000009ffff
    [ 0.259547] reserve RAM buffer: 00000000cff70000 - 00000000cfffffff
    [ 0.259624] NetLabel: Initializing
    [ 0.259626] NetLabel: domain hash size = 128
    [ 0.259627] NetLabel: protocols = UNLABELED CIPSOv4
    [ 0.259637] NetLabel: unlabeled traffic allowed by default
    [ 0.259649] HPET: 4 timers in total, 0 timers will be used for per-cpu timer
    [ 0.259653] hpet0: at MMIO 0xfed00000, IRQs 2, 8, 0, 0
    [ 0.259656] hpet0: 4 comparators, 64-bit 14.318180 MHz counter
    [ 0.270027] Switching to clocksource hpet
    [ 0.274998] pnp: PnP ACPI init
    [ 0.275013] ACPI: bus type pnp registered
    [ 0.275071] pnp 00:00: [bus 00-ff]
    [ 0.275073] pnp 00:00: [io 0x0cf8-0x0cff]
    [ 0.275074] pnp 00:00: [io 0x0000-0x0cf7 window]
    [ 0.275076] pnp 00:00: [io 0x0d00-0xffff window]
    [ 0.275078] pnp 00:00: [mem 0x000a0000-0x000bffff window]
    [ 0.275079] pnp 00:00: [mem 0x000d0000-0x000dffff window]
    [ 0.275081] pnp 00:00: [mem 0xd0000000-0xffffffff window]
    [ 0.275124] pnp 00:00: Plug and Play ACPI device, IDs PNP0a08 PNP0a03 (active)
    [ 0.275131] pnp 00:01: [mem 0xfed14000-0xfed19fff]
    [ 0.275167] system 00:01: [mem 0xfed14000-0xfed19fff] has been reserved
    [ 0.275170] system 00:01: Plug and Play ACPI device, IDs PNP0c01 (active)
    [ 0.275193] pnp 00:02: [dma 4]
    [ 0.275194] pnp 00:02: [io 0x0000-0x000f]
    [ 0.275196] pnp 00:02: [io 0x0081-0x0083]
    [ 0.275197] pnp 00:02: [io 0x0087]
    [ 0.275198] pnp 00:02: [io 0x0089-0x008b]
    [ 0.275202] pnp 00:02: [io 0x008f]
    [ 0.275203] pnp 00:02: [io 0x00c0-0x00df]
    [ 0.275225] pnp 00:02: Plug and Play ACPI device, IDs PNP0200 (active)
    [ 0.275234] pnp 00:03: [io 0x0070-0x0071]
    [ 0.275243] pnp 00:03: [irq 8]
    [ 0.275263] pnp 00:03: Plug and Play ACPI device, IDs PNP0b00 (active)
    [ 0.275269] pnp 00:04: [io 0x0061]
    [ 0.275290] pnp 00:04: Plug and Play ACPI device, IDs PNP0800 (active)
    [ 0.275296] pnp 00:05: [io 0x00f0-0x00ff]
    [ 0.275300] pnp 00:05: [irq 13]
    [ 0.275322] pnp 00:05: Plug and Play ACPI device, IDs PNP0c04 (active)
    [ 0.275431] pnp 00:06: [io 0x0000-0xffffffffffffffff disabled]
    [ 0.275433] pnp 00:06: [io 0x0000-0xffffffffffffffff disabled]
    [ 0.275435] pnp 00:06: [io 0x0290-0x029f]
    [ 0.275467] system 00:06: [io 0x0290-0x029f] has been reserved
    [ 0.275470] system 00:06: Plug and Play ACPI device, IDs PNP0c02 (active)
    [ 0.275531] pnp 00:07: [io 0x0010-0x001f]
    [ 0.275533] pnp 00:07: [io 0x0022-0x003f]
    [ 0.275534] pnp 00:07: [io 0x0044-0x004d]
    [ 0.275536] pnp 00:07: [io 0x0050-0x005f]
    [ 0.275537] pnp 00:07: [io 0x0062-0x0063]
    [ 0.275538] pnp 00:07: [io 0x0065-0x006f]
    [ 0.275540] pnp 00:07: [io 0x0072-0x007f]
    [ 0.275541] pnp 00:07: [io 0x0080]
    [ 0.275542] pnp 00:07: [io 0x0084-0x0086]
    [ 0.275543] pnp 00:07: [io 0x0088]
    [ 0.275545] pnp 00:07: [io 0x008c-0x008e]
    [ 0.275546] pnp 00:07: [io 0x0090-0x009f]
    [ 0.275547] pnp 00:07: [io 0x00a2-0x00bf]
    [ 0.275549] pnp 00:07: [io 0x00e0-0x00ef]
    [ 0.275550] pnp 00:07: [io 0x04d0-0x04d1]
    [ 0.275551] pnp 00:07: [io 0x0800-0x087f]
    [ 0.275553] pnp 00:07: [io 0x0400-0x03ff disabled]
    [ 0.275554] pnp 00:07: [io 0x0500-0x057f]
    [ 0.275556] pnp 00:07: [mem 0xfed08000-0xfed08fff]
    [ 0.275557] pnp 00:07: [mem 0xfed1c000-0xfed1ffff]
    [ 0.275558] pnp 00:07: [mem 0xfed20000-0xfed3ffff]
    [ 0.275560] pnp 00:07: [mem 0xfed50000-0xfed8ffff]
    [ 0.275610] system 00:07: [io 0x04d0-0x04d1] has been reserved
    [ 0.275612] system 00:07: [io 0x0800-0x087f] has been reserved
    [ 0.275614] system 00:07: [io 0x0500-0x057f] has been reserved
    [ 0.275616] system 00:07: [mem 0xfed08000-0xfed08fff] has been reserved
    [ 0.275618] system 00:07: [mem 0xfed1c000-0xfed1ffff] has been reserved
    [ 0.275620] system 00:07: [mem 0xfed20000-0xfed3ffff] has been reserved
    [ 0.275622] system 00:07: [mem 0xfed50000-0xfed8ffff] has been reserved
    [ 0.275625] system 00:07: Plug and Play ACPI device, IDs PNP0c02 (active)
    [ 0.275666] pnp 00:08: [mem 0xfed00000-0xfed003ff]
    [ 0.275690] pnp 00:08: Plug and Play ACPI device, IDs PNP0103 (active)
    [ 0.275722] pnp 00:09: [mem 0xffb00000-0xffbfffff]
    [ 0.275724] pnp 00:09: [mem 0xfff00000-0xffffffff]
    [ 0.275745] pnp 00:09: Plug and Play ACPI device, IDs INT0800 (active)
    [ 0.275772] pnp 00:0a: [mem 0xffc00000-0xffefffff]
    [ 0.275807] system 00:0a: [mem 0xffc00000-0xffefffff] has been reserved
    [ 0.275809] system 00:0a: Plug and Play ACPI device, IDs PNP0c02 (active)
    [ 0.275845] pnp 00:0b: [io 0x0000-0xffffffffffffffff disabled]
    [ 0.275847] pnp 00:0b: [io 0x0000-0xffffffffffffffff disabled]
    [ 0.275848] pnp 00:0b: [mem 0xfec00000-0xfec00fff]
    [ 0.275850] pnp 00:0b: [mem 0xfee00000-0xfee00fff]
    [ 0.275885] system 00:0b: [mem 0xfec00000-0xfec00fff] could not be reserved
    [ 0.275887] system 00:0b: [mem 0xfee00000-0xfee00fff] has been reserved
    [ 0.275889] system 00:0b: Plug and Play ACPI device, IDs PNP0c02 (active)
    [ 0.275908] pnp 00:0c: [io 0x0060]
    [ 0.275909] pnp 00:0c: [io 0x0064]
    [ 0.275915] pnp 00:0c: [irq 1]
    [ 0.275945] pnp 00:0c: Plug and Play ACPI device, IDs PNP0303 PNP030b (active)
    [ 0.276048] pnp 00:0d: [mem 0xe0000000-0xefffffff]
    [ 0.276083] system 00:0d: [mem 0xe0000000-0xefffffff] has been reserved
    [ 0.276086] system 00:0d: Plug and Play ACPI device, IDs PNP0c02 (active)
    [ 0.276183] pnp 00:0e: [mem 0x00000000-0x0009ffff]
    [ 0.276185] pnp 00:0e: [mem 0x000c0000-0x000cffff]
    [ 0.276187] pnp 00:0e: [mem 0x000e0000-0x000fffff]
    [ 0.276188] pnp 00:0e: [mem 0x00100000-0xcfffffff]
    [ 0.276190] pnp 00:0e: [mem 0x00000000-0xffffffffffffffff disabled]
    [ 0.276230] system 00:0e: [mem 0x00000000-0x0009ffff] could not be reserved
    [ 0.276232] system 00:0e: [mem 0x000c0000-0x000cffff] could not be reserved
    [ 0.276234] system 00:0e: [mem 0x000e0000-0x000fffff] could not be reserved
    [ 0.276236] system 00:0e: [mem 0x00100000-0xcfffffff] could not be reserved
    [ 0.276238] system 00:0e: Plug and Play ACPI device, IDs PNP0c01 (active)
    [ 0.276322] pnp: PnP ACPI: found 15 devices
    [ 0.276323] ACPI: ACPI bus type pnp unregistered
    [ 0.282821] PCI: max bus depth: 1 pci_try_num: 2
    [ 0.282845] pci 0000:00:1c.5: BAR 15: assigned [mem 0xf0000000-0xf01fffff 64bit pref]
    [ 0.282848] pci 0000:00:1c.0: BAR 14: assigned [mem 0xf0200000-0xf05fffff]
    [ 0.282851] pci 0000:00:1c.0: BAR 13: assigned [io 0x1000-0x1fff]
    [ 0.282853] pci 0000:00:01.0: PCI bridge to [bus 01-01]
    [ 0.282855] pci 0000:00:01.0: bridge window [io 0xd000-0xdfff]
    [ 0.282857] pci 0000:00:01.0: bridge window [mem 0xfa000000-0xfeafffff]
    [ 0.282860] pci 0000:00:01.0: bridge window [mem 0xd0000000-0xdfffffff 64bit pref]
    [ 0.282863] pci 0000:00:1c.0: PCI bridge to [bus 03-03]
    [ 0.282865] pci 0000:00:1c.0: bridge window [io 0x1000-0x1fff]
    [ 0.282869] pci 0000:00:1c.0: bridge window [mem 0xf0200000-0xf05fffff]
    [ 0.282872] pci 0000:00:1c.0: bridge window [mem 0xf8f00000-0xf8ffffff 64bit pref]
    [ 0.282877] pci 0000:00:1c.5: PCI bridge to [bus 02-02]
    [ 0.282879] pci 0000:00:1c.5: bridge window [io 0xe000-0xefff]
    [ 0.282882] pci 0000:00:1c.5: bridge window [mem 0xfeb00000-0xfebfffff]
    [ 0.282885] pci 0000:00:1c.5: bridge window [mem 0xf0000000-0xf01fffff 64bit pref]
    [ 0.282890] pci 0000:00:1e.0: PCI bridge to [bus 04-04]
    [ 0.282906] pci 0000:00:01.0: PCI INT A -> GSI 16 (level, low) -> IRQ 16
    [ 0.282909] pci 0000:00:01.0: setting latency timer to 64
    [ 0.282913] pci 0000:00:1c.0: enabling device (0106 -> 0107)
    [ 0.282919] pci 0000:00:1c.0: PCI INT A -> GSI 17 (level, low) -> IRQ 17
    [ 0.282922] pci 0000:00:1c.0: setting latency timer to 64
    [ 0.282926] pci 0000:00:1c.5: PCI INT B -> GSI 16 (level, low) -> IRQ 16
    [ 0.282929] pci 0000:00:1c.5: setting latency timer to 64
    [ 0.282934] pci 0000:00:1e.0: setting latency timer to 64
    [ 0.282937] pci_bus 0000:00: resource 4 [io 0x0000-0x0cf7]
    [ 0.282938] pci_bus 0000:00: resource 5 [io 0x0d00-0xffff]
    [ 0.282940] pci_bus 0000:00: resource 6 [mem 0x000a0000-0x000bffff]
    [ 0.282942] pci_bus 0000:00: resource 7 [mem 0x000d0000-0x000dffff]
    [ 0.282943] pci_bus 0000:00: resource 8 [mem 0xd0000000-0xffffffff]
    [ 0.282945] pci_bus 0000:01: resource 0 [io 0xd000-0xdfff]
    [ 0.282947] pci_bus 0000:01: resource 1 [mem 0xfa000000-0xfeafffff]
    [ 0.282949] pci_bus 0000:01: resource 2 [mem 0xd0000000-0xdfffffff 64bit pref]
    [ 0.282950] pci_bus 0000:03: resource 0 [io 0x1000-0x1fff]
    [ 0.282952] pci_bus 0000:03: resource 1 [mem 0xf0200000-0xf05fffff]
    [ 0.282954] pci_bus 0000:03: resource 2 [mem 0xf8f00000-0xf8ffffff 64bit pref]
    [ 0.282956] pci_bus 0000:02: resource 0 [io 0xe000-0xefff]
    [ 0.282957] pci_bus 0000:02: resource 1 [mem 0xfeb00000-0xfebfffff]
    [ 0.282959] pci_bus 0000:02: resource 2 [mem 0xf0000000-0xf01fffff 64bit pref]
    [ 0.282961] pci_bus 0000:04: resource 4 [io 0x0000-0x0cf7]
    [ 0.282962] pci_bus 0000:04: resource 5 [io 0x0d00-0xffff]
    [ 0.282964] pci_bus 0000:04: resource 6 [mem 0x000a0000-0x000bffff]
    [ 0.282966] pci_bus 0000:04: resource 7 [mem 0x000d0000-0x000dffff]
    [ 0.282967] pci_bus 0000:04: resource 8 [mem 0xd0000000-0xffffffff]
    [ 0.282994] NET: Registered protocol family 2
    [ 0.283102] IP route cache hash table entries: 131072 (order: 8, 1048576 bytes)
    [ 0.283911] TCP established hash table entries: 524288 (order: 11, 8388608 bytes)
    [ 0.286830] TCP bind hash table entries: 65536 (order: 8, 1048576 bytes)
    [ 0.287221] TCP: Hash tables configured (established 524288 bind 65536)
    [ 0.287223] TCP reno registered
    [ 0.287231] UDP hash table entries: 2048 (order: 4, 65536 bytes)
    [ 0.287261] UDP-Lite hash table entries: 2048 (order: 4, 65536 bytes)
    [ 0.287380] NET: Registered protocol family 1
    [ 0.287533] pci 0000:01:00.0: Boot video device
    [ 0.287538] PCI: CLS 32 bytes, default 64
    [ 0.287587] Unpacking initramfs...
    [ 0.316854] Freeing initrd memory: 2536k freed
    [ 0.317344] PCI-DMA: Using software bounce buffering for IO (SWIOTLB)
    [ 0.317346] Placing 64MB software IO TLB between ffff8800cbf6a000 - ffff8800cff6a000
    [ 0.317348] software IO TLB at phys 0xcbf6a000 - 0xcff6a000
    [ 0.317656] audit: initializing netlink socket (disabled)
    [ 0.317668] type=2000 audit(1331070636.316:1): initialized
    [ 0.329229] HugeTLB registered 2 MB page size, pre-allocated 0 pages
    [ 0.351410] VFS: Disk quotas dquot_6.5.2
    [ 0.351460] Dquot-cache hash table entries: 512 (order 0, 4096 bytes)
    [ 0.351537] msgmni has been set to 7910
    [ 0.351674] Block layer SCSI generic (bsg) driver version 0.4 loaded (major 253)
    [ 0.351694] io scheduler noop registered
    [ 0.351696] io scheduler deadline registered
    [ 0.351723] io scheduler cfq registered (default)
    [ 0.351807] pcieport 0000:00:01.0: setting latency timer to 64
    [ 0.351832] pcieport 0000:00:01.0: irq 40 for MSI/MSI-X
    [ 0.351871] pcieport 0000:00:1c.0: setting latency timer to 64
    [ 0.351899] pcieport 0000:00:1c.0: irq 41 for MSI/MSI-X
    [ 0.351947] pcieport 0000:00:1c.5: setting latency timer to 64
    [ 0.351975] pcieport 0000:00:1c.5: irq 42 for MSI/MSI-X
    [ 0.352104] intel_idle: MWAIT substates: 0x22220
    [ 0.352105] intel_idle: does not run on family 6 model 23
    [ 0.352132] ERST: Table is not found!
    [ 0.352133] GHES: HEST is not enabled!
    [ 0.352190] Serial: 8250/16550 driver, 4 ports, IRQ sharing disabled
    [ 0.423563] Linux agpgart interface v0.103
    [ 0.423624] i8042: PNP: PS/2 Controller [PNP0303:PS2K] at 0x60,0x64 irq 1
    [ 0.423626] i8042: PNP: PS/2 appears to have AUX port disabled, if this is incorrect please boot with i8042.nopnp
    [ 0.424352] serio: i8042 KBD port at 0x60,0x64 irq 1
    [ 0.424437] mousedev: PS/2 mouse device common for all mice
    [ 0.424487] rtc_cmos 00:03: RTC can wake from S4
    [ 0.424570] rtc_cmos 00:03: rtc core: registered rtc_cmos as rtc0
    [ 0.424591] rtc0: alarms up to one month, y3k, 114 bytes nvram, hpet irqs
    [ 0.424600] cpuidle: using governor ladder
    [ 0.424601] cpuidle: using governor menu
    [ 0.424761] TCP cubic registered
    [ 0.424857] NET: Registered protocol family 10
    [ 0.425184] NET: Registered protocol family 17
    [ 0.425187] Registering the dns_resolver key type
    [ 0.425287] PM: Checking hibernation image partition /dev/disk/by-uuid/5a94d96e-0fdd-425a-91c5-6fe96da349f0
    [ 0.442444] input: AT Translated Set 2 keyboard as /devices/platform/i8042/serio0/input/input0
    [ 0.442457] PM: Hibernation image not present or could not be loaded.
    [ 0.442463] registered taskstats version 1
    [ 0.453068] rtc_cmos 00:03: setting system clock to 2012-03-06 21:50:37 UTC (1331070637)
    [ 0.453104] Initializing network drop monitor service
    [ 0.454145] Freeing unused kernel memory: 736k freed
    [ 0.454268] Write protecting the kernel read-only data: 8192k
    [ 0.458421] Freeing unused kernel memory: 1636k freed
    [ 0.460355] Freeing unused kernel memory: 660k freed
    [ 0.465585] udevd[37]: starting version 181
    [ 0.489348] usbcore: registered new interface driver usbfs
    [ 0.489369] usbcore: registered new interface driver hub
    [ 0.490707] usbcore: registered new device driver usb
    [ 0.492744] ehci_hcd: USB 2.0 'Enhanced' Host Controller (EHCI) Driver
    [ 0.492777] ehci_hcd 0000:00:1a.7: PCI INT C -> GSI 18 (level, low) -> IRQ 18
    [ 0.492811] ehci_hcd 0000:00:1a.7: setting latency timer to 64
    [ 0.492815] ehci_hcd 0000:00:1a.7: EHCI Host Controller
    [ 0.492852] ehci_hcd 0000:00:1a.7: new USB bus registered, assigned bus number 1
    [ 0.492879] ehci_hcd 0000:00:1a.7: debug port 1
    [ 0.497714] uhci_hcd: USB Universal Host Controller Interface driver
    [ 0.497902] ehci_hcd 0000:00:1a.7: cache line size of 32 is not supported
    [ 0.497918] ehci_hcd 0000:00:1a.7: irq 18, io mem 0xf9fffc00
    [ 0.500931] SCSI subsystem initialized
    [ 0.504356] libata version 3.00 loaded.
    [ 0.510015] ehci_hcd 0000:00:1a.7: USB 2.0 started, EHCI 1.00
    [ 0.510276] hub 1-0:1.0: USB hub found
    [ 0.510280] hub 1-0:1.0: 6 ports detected
    [ 0.510367] ehci_hcd 0000:00:1d.7: PCI INT A -> GSI 23 (level, low) -> IRQ 23
    [ 0.510390] ehci_hcd 0000:00:1d.7: setting latency timer to 64
    [ 0.510393] ehci_hcd 0000:00:1d.7: EHCI Host Controller
    [ 0.510401] ehci_hcd 0000:00:1d.7: new USB bus registered, assigned bus number 2
    [ 0.510423] ehci_hcd 0000:00:1d.7: debug port 1
    [ 0.514289] ehci_hcd 0000:00:1d.7: cache line size of 32 is not supported
    [ 0.514303] ehci_hcd 0000:00:1d.7: irq 23, io mem 0xf9fff800
    [ 0.526698] ehci_hcd 0000:00:1d.7: USB 2.0 started, EHCI 1.00
    [ 0.526921] hub 2-0:1.0: USB hub found
    [ 0.526924] hub 2-0:1.0: 6 ports detected
    [ 0.527005] uhci_hcd 0000:00:1a.0: PCI INT A -> GSI 16 (level, low) -> IRQ 16
    [ 0.527010] uhci_hcd 0000:00:1a.0: setting latency timer to 64
    [ 0.527013] uhci_hcd 0000:00:1a.0: UHCI Host Controller
    [ 0.527021] uhci_hcd 0000:00:1a.0: new USB bus registered, assigned bus number 3
    [ 0.527047] uhci_hcd 0000:00:1a.0: irq 16, io base 0x0000c800
    [ 0.527239] hub 3-0:1.0: USB hub found
    [ 0.527243] hub 3-0:1.0: 2 ports detected
    [ 0.527303] uhci_hcd 0000:00:1a.1: PCI INT B -> GSI 21 (level, low) -> IRQ 21
    [ 0.527308] uhci_hcd 0000:00:1a.1: setting latency timer to 64
    [ 0.527310] uhci_hcd 0000:00:1a.1: UHCI Host Controller
    [ 0.527316] uhci_hcd 0000:00:1a.1: new USB bus registered, assigned bus number 4
    [ 0.527341] uhci_hcd 0000:00:1a.1: irq 21, io base 0x0000c880
    [ 0.527491] hub 4-0:1.0: USB hub found
    [ 0.527494] hub 4-0:1.0: 2 ports detected
    [ 0.527547] uhci_hcd 0000:00:1a.2: PCI INT C -> GSI 18 (level, low) -> IRQ 18
    [ 0.527552] uhci_hcd 0000:00:1a.2: setting latency timer to 64
    [ 0.527554] uhci_hcd 0000:00:1a.2: UHCI Host Controller
    [ 0.527560] uhci_hcd 0000:00:1a.2: new USB bus registered, assigned bus number 5
    [ 0.527579] uhci_hcd 0000:00:1a.2: irq 18, io base 0x0000cc00
    [ 0.527671] hub 5-0:1.0: USB hub found
    [ 0.527673] hub 5-0:1.0: 2 ports detected
    [ 0.527725] uhci_hcd 0000:00:1d.0: PCI INT A -> GSI 23 (level, low) -> IRQ 23
    [ 0.527729] uhci_hcd 0000:00:1d.0: setting latency timer to 64
    [ 0.527731] uhci_hcd 0000:00:1d.0: UHCI Host Controller
    [ 0.527737] uhci_hcd 0000:00:1d.0: new USB bus registered, assigned bus number 6
    [ 0.527755] uhci_hcd 0000:00:1d.0: irq 23, io base 0x0000c080
    [ 0.527850] hub 6-0:1.0: USB hub found
    [ 0.527853] hub 6-0:1.0: 2 ports detected
    [ 0.527912] uhci_hcd 0000:00:1d.1: PCI INT B -> GSI 19 (level, low) -> IRQ 19
    [ 0.527916] uhci_hcd 0000:00:1d.1: setting latency timer to 64
    [ 0.527918] uhci_hcd 0000:00:1d.1: UHCI Host Controller
    [ 0.527925] uhci_hcd 0000:00:1d.1: new USB bus registered, assigned bus number 7
    [ 0.527950] uhci_hcd 0000:00:1d.1: irq 19, io base 0x0000c400
    [ 0.528041] hub 7-0:1.0: USB hub found
    [ 0.528044] hub 7-0:1.0: 2 ports detected
    [ 0.528095] uhci_hcd 0000:00:1d.2: PCI INT C -> GSI 18 (level, low) -> IRQ 18
    [ 0.528099] uhci_hcd 0000:00:1d.2: setting latency timer to 64
    [ 0.528102] uhci_hcd 0000:00:1d.2: UHCI Host Controller
    [ 0.528107] uhci_hcd 0000:00:1d.2: new USB bus registered, assigned bus number 8
    [ 0.528126] uhci_hcd 0000:00:1d.2: irq 18, io base 0x0000c480
    [ 0.528217] hub 8-0:1.0: USB hub found
    [ 0.528220] hub 8-0:1.0: 2 ports detected
    [ 0.528277] ata_piix 0000:00:1f.2: version 2.13
    [ 0.528284] ata_piix 0000:00:1f.2: PCI INT B -> GSI 19 (level, low) -> IRQ 19
    [ 0.528288] ata_piix 0000:00:1f.2: MAP [ P0 P2 P1 P3 ]
    [ 0.528322] ata_piix 0000:00:1f.2: setting latency timer to 64
    [ 0.528959] scsi0 : ata_piix
    [ 0.529039] scsi1 : ata_piix
    [ 0.529777] ata1: SATA max UDMA/133 cmd 0xb000 ctl 0xac00 bmdma 0xa480 irq 19
    [ 0.529781] ata2: SATA max UDMA/133 cmd 0xa880 ctl 0xa800 bmdma 0xa488 irq 19
    [ 0.529808] ata_piix 0000:00:1f.5: PCI INT B -> GSI 19 (level, low) -> IRQ 19
    [ 0.529812] ata_piix 0000:00:1f.5: MAP [ P0 -- P1 -- ]
    [ 0.529848] ata_piix 0000:00:1f.5: setting latency timer to 64
    [ 0.530605] scsi2 : ata_piix
    [ 0.531122] scsi3 : ata_piix
    [ 0.531716] ata3: SATA max UDMA/133 cmd 0xc000 ctl 0xbc00 bmdma 0xb480 irq 19
    [ 0.531719] ata4: SATA max UDMA/133 cmd 0xb880 ctl 0xb800 bmdma 0xb488 irq 19
    [ 0.833346] usb 2-1: new high-speed USB device number 2 using ehci_hcd
    [ 0.860625] ata3: SATA link down (SStatus 0 SControl 300)
    [ 0.871253] ata4: SATA link down (SStatus 0 SControl 300)
    [ 1.133351] usb 2-5: new high-speed USB device number 4 using ehci_hcd
    [ 1.320013] Refined TSC clocksource calibration: 2996.999 MHz.
    [ 1.320019] Switching to clocksource tsc
    [ 1.326723] ata2.00: SATA link up 3.0 Gbps (SStatus 123 SControl 300)
    [ 1.326735] ata2.01: SATA link up 3.0 Gbps (SStatus 123 SControl 300)
    [ 1.326848] ata1.00: SATA link up 1.5 Gbps (SStatus 113 SControl 300)
    [ 1.326858] ata1.01: SATA link down (SStatus 0 SControl 300)
    [ 1.334172] ata1.00: ATA-7: WDC WD740ADFD-00NLR1, 20.07P20, max UDMA/133
    [ 1.334175] ata1.00: 145226112 sectors, multi 16: LBA48 NCQ (not used)
    [ 1.347438] ata1.00: configured for UDMA/133
    [ 1.347545] scsi 0:0:0:0: Direct-Access ATA WDC WD740ADFD-00 20.0 PQ: 0 ANSI: 5
    [ 1.356883] ata2.00: ATA-7: SAMSUNG HD321KJ, CP100-05, max UDMA7
    [ 1.356887] ata2.00: 625142448 sectors, multi 16: LBA48 NCQ (depth 0/32)
    [ 1.371850] ata2.01: HPA detected: current 1465147055, native 1465149168
    [ 1.371855] ata2.01: ATA-8: WDC WD7500AACS-00ZJB0, 01.01B01, max UDMA/133
    [ 1.371858] ata2.01: 1465147055 sectors, multi 16: LBA48 NCQ (depth 0/32)
    [ 1.376886] ata2.00: configured for UDMA/133
    [ 1.390777] ata2.01: configured for UDMA/133
    [ 1.390856] scsi 1:0:0:0: Direct-Access ATA SAMSUNG HD321KJ CP10 PQ: 0 ANSI: 5
    [ 1.390990] scsi 1:0:1:0: Direct-Access ATA WDC WD7500AACS-0 01.0 PQ: 0 ANSI: 5
    [ 1.393373] sd 0:0:0:0: [sda] 145226112 512-byte logical blocks: (74.3 GB/69.2 GiB)
    [ 1.393409] sd 0:0:0:0: [sda] Write Protect is off
    [ 1.393411] sd 0:0:0:0: [sda] Mode Sense: 00 3a 00 00
    [ 1.393427] sd 0:0:0:0: [sda] Write cache: enabled, read cache: enabled, doesn't support DPO or FUA
    [ 1.393753] sd 1:0:0:0: [sdb] 625142448 512-byte logical blocks: (320 GB/298 GiB)
    [ 1.393840] sd 1:0:0:0: [sdb] Write Protect is off
    [ 1.393843] sd 1:0:0:0: [sdb] Mode Sense: 00 3a 00 00
    [ 1.393860] sd 1:0:0:0: [sdb] Write cache: enabled, read cache: enabled, doesn't support DPO or FUA
    [ 1.394066] sd 1:0:1:0: [sdc] 1465147055 512-byte logical blocks: (750 GB/698 GiB)
    [ 1.404247] sdb: sdb2
    [ 1.404286] sd 1:0:1:0: [sdc] Write Protect is off
    [ 1.404290] sd 1:0:1:0: [sdc] Mode Sense: 00 3a 00 00
    [ 1.404316] sd 1:0:1:0: [sdc] Write cache: enabled, read cache: enabled, doesn't support DPO or FUA
    [ 1.410206] sd 1:0:0:0: [sdb] Attached SCSI disk
    [ 1.410713] sdc: sdc1
    [ 1.412675] sd 1:0:1:0: [sdc] Attached SCSI disk
    [ 1.414632] sda: sda1 sda2 sda3
    [ 1.414890] sd 0:0:0:0: [sda] Attached SCSI disk
    [ 1.483346] usb 7-1: new low-speed USB device number 2 using uhci_hcd
    [ 1.675206] input: Logitech USB-PS/2 Optical Mouse as /devices/pci0000:00/0000:00:1d.1/usb7/7-1/7-1:1.0/input/input1
    [ 1.675328] generic-usb 0003:046D:C01E.0001: input,hidraw0: USB HID v1.10 Mouse [Logitech USB-PS/2 Optical Mouse] on usb-0000:00:1d.1-1/input0
    [ 1.675340] usbcore: registered new interface driver usbhid
    [ 1.675341] usbhid: USB HID core driver
    [ 1.847718] PM: Starting manual resume from disk
    [ 1.847720] PM: Hibernation image partition 8:2 present
    [ 1.847721] PM: Looking for hibernation image.
    [ 1.847851] PM: Image not found (code -22)
    [ 1.847854] PM: Hibernation image not present or could not be loaded.
    [ 1.887347] EXT4-fs (sda1): mounted filesystem with ordered data mode. Opts: (null)
    [ 2.955372] udevd[142]: starting version 181
    [ 3.102653] FS-Cache: Loaded
    [ 3.138386] RPC: Registered named UNIX socket transport module.
    [ 3.138388] RPC: Registered udp transport module.
    [ 3.138390] RPC: Registered tcp transport module.
    [ 3.138391] RPC: Registered tcp NFSv4.1 backchannel transport module.
    [ 3.206448] FS-Cache: Netfs 'nfs' registered for caching
    [ 3.340426] input: Power Button as /devices/LNXSYSTM:00/device:00/PNP0C0C:00/input/input2
    [ 3.340433] ACPI: Power Button [PWRB]
    [ 3.340480] input: Power Button as /devices/LNXSYSTM:00/LNXPWRBN:00/input/input3
    [ 3.340483] ACPI: Power Button [PWRF]
    [ 3.367741] iTCO_vendor_support: vendor-support=0
    [ 3.368037] i801_smbus 0000:00:1f.3: PCI INT C -> GSI 18 (level, low) -> IRQ 18
    [ 3.368042] ACPI: resource 0000:00:1f.3 [io 0x0400-0x041f] conflicts with ACPI region SMRG [io 0x400-0x40f]
    [ 3.368043] ACPI: If an ACPI driver is available for this device, you should use it instead of the native driver
    [ 3.420207] ATL1E 0000:02:00.0: PCI INT A -> GSI 17 (level, low) -> IRQ 17
    [ 3.420218] ATL1E 0000:02:00.0: setting latency timer to 64
    [ 3.460984] input: PC Speaker as /devices/platform/pcspkr/input/input4
    [ 3.646986] nvidia: module license 'NVIDIA' taints kernel.
    [ 3.646990] Disabling lock debugging due to kernel taint
    [ 3.658130] iTCO_wdt: Intel TCO WatchDog Timer Driver v1.07
    [ 3.658258] iTCO_wdt: Found a ICH10 TCO device (Version=2, TCOBASE=0x0860)
    [ 3.658557] iTCO_wdt: initialized. heartbeat=30 sec (nowayout=0)
    [ 3.808444] nvidia 0000:01:00.0: PCI INT A -> GSI 16 (level, low) -> IRQ 16
    [ 3.808451] nvidia 0000:01:00.0: setting latency timer to 64
    [ 3.808456] vgaarb: device changed decodes: PCI:0000:01:00.0,olddecodes=io+mem,decodes=none:owns=io+mem
    [ 3.808546] NVRM: loading NVIDIA UNIX x86_64 Kernel Module 295.20 Mon Feb 6 21:07:30 PST 2012
    [ 3.922194] snd_hda_intel 0000:00:1b.0: PCI INT A -> GSI 22 (level, low) -> IRQ 22
    [ 3.922239] snd_hda_intel 0000:00:1b.0: irq 43 for MSI/MSI-X
    [ 3.922258] snd_hda_intel 0000:00:1b.0: setting latency timer to 64
    [ 3.964505] input: HDA Digital PCBeep as /devices/pci0000:00/0000:00:1b.0/input/input5
    [ 4.187852] cfg80211: Calling CRDA to update world regulatory domain
    [ 4.420014] usb 2-1: reset high-speed USB device number 2 using ehci_hcd
    [ 4.487427] usblp0: USB Bidirectional printer dev 4 if 0 alt 0 proto 2 vid 0x04A9 pid 0x10B6
    [ 4.487464] usbcore: registered new interface driver usblp
    [ 4.597153] ieee80211 phy0: Selected rate control algorithm 'minstrel_ht'
    [ 4.597522] Registered led device: rt2800usb-phy0::radio
    [ 4.597538] Registered led device: rt2800usb-phy0::assoc
    [ 4.597553] Registered led device: rt2800usb-phy0::quality
    [ 4.597572] usbcore: registered new interface driver rt2800usb
    [ 4.896528] EXT4-fs (sda1): re-mounted. Opts: (null)
    [ 4.931213] EXT4-fs (sda3): mounted filesystem with ordered data mode. Opts: (null)
    [ 4.962285] EXT4-fs (sdc1): mounted filesystem with ordered data mode. Opts: (null)
    [ 5.004187] Adding 8279036k swap on /dev/sda2. Priority:-1 extents:1 across:8279036k
    [ 5.887306] Installing knfsd (copyright (C) 1996 [email protected]).
    [ 36.076486] ATL1E 0000:02:00.0: irq 44 for MSI/MSI-X
    [ 36.077062] ADDRCONF(NETDEV_UP): eth0: link is not ready
    [ 50.139089] ADDRCONF(NETDEV_UP): wlan0: link is not ready
    [ 53.445833] ADDRCONF(NETDEV_UP): wlan0: link is not ready
    [ 53.490782] ATL1E 0000:02:00.0: irq 44 for MSI/MSI-X
    [ 53.491333] ADDRCONF(NETDEV_UP): eth0: link is not ready
    [ 53.732458] ADDRCONF(NETDEV_UP): wlan0: link is not ready
    [ 57.151881] wlan0: authenticate with c0:c1:c0:36:e7:5d (try 1)
    [ 57.153391] wlan0: authenticated
    [ 57.170628] wlan0: associate with c0:c1:c0:36:e7:5d (try 1)
    [ 57.174138] wlan0: RX AssocResp from c0:c1:c0:36:e7:5d (capab=0x411 status=0 aid=1)
    [ 57.174142] wlan0: associated
    [ 57.197936] ADDRCONF(NETDEV_CHANGE): wlan0: link becomes ready
    [ 57.264846] Intel AES-NI instructions are not detected.
    [ 67.456668] wlan0: no IPv6 routers present
    [ 306.299972] ATL1E 0000:02:00.0: eth0: NIC Link is Up <100 Mbps Full Duplex>
    [ 306.300485] ADDRCONF(NETDEV_CHANGE): eth0: link becomes ready
    [ 310.963856] wlan0: deauthenticating from c0:c1:c0:36:e7:5d by local choice (reason=3)
    [ 311.091991] cfg80211: Calling CRDA to update world regulatory domain
    [ 311.342430] ADDRCONF(NETDEV_UP): wlan0: link is not ready
    [ 313.239060] ADDRCONF(NETDEV_UP): wlan0: link is not ready
    [ 314.375684] ADDRCONF(NETDEV_UP): wlan0: link is not ready
    [ 314.545703] ATL1E 0000:02:00.0: irq 44 for MSI/MSI-X
    [ 314.545818] ATL1E 0000:02:00.0: eth0: NIC Link is Up <100 Mbps Full Duplex>
    [ 314.581962] ATL1E 0000:02:00.0: irq 44 for MSI/MSI-X
    [ 314.582102] ATL1E 0000:02:00.0: eth0: NIC Link is Up <100 Mbps Full Duplex>
    [ 314.582703] ADDRCONF(NETDEV_UP): eth0: link is not ready
    [ 314.583112] ADDRCONF(NETDEV_CHANGE): eth0: link becomes ready
    [ 325.469976] eth0: no IPv6 routers present
    Last edited by dabbi2000 (2012-03-07 22:57:16)

    certainly didn't expect this... turned off power saving
    iwconfig wlan0 power off
    and all is well again. Surely doesn't explain why it started after kernel update. But it works...

Maybe you are looking for